Downtown Meaning Main City: The Proposed Railway Will Supplant The Traffic Lanes of Several Crucial City Highways
Downtown Meaning Main City: The Proposed Railway Will Supplant The Traffic Lanes of Several Crucial City Highways
html
http://gmatclub.com/forum/100-hardest-critical-reasoning-questions-ver-174349.html
Also, eliminate the answers that undermine, rather than support, the mayor's plan.
The mayor's plan to reduce city traffic congestion is to construct a railway commuter system, stretching comprehensively outwards
from the downtown area and ending at the city limits. According to the mayor, construction of the railway system would cost less
than adding new traffic lanes and would reduce the amount of car traffic in the city.
Question
Which of the following, if true, could the mayor present as evidence to support her plan?
Choices
A
The proposed railway will supplant the traffic lanes of several crucial city
highways.
This is not the credited choice. Since the new railway reduces city highway access, it undermines the mayor's plan to reduce car
traffic.
B
A lot of city traffic congestion is caused by incoming truck shipments from remote locations outside the city.
This is not the credited choice because it undermines the mayor's plan. If a lot of traffic on the city roads is due to trucks from
outside the city, the mayor's plan is likely to reduce city traffic since the trucks won't be using rail.
C
For most city dwellers, the commuter railway fares will be more expensive than the cost of commuting by car.
This is not the credited choice. It undermines the mayor's plan because expensive railway fares will dissuade commuters.
D
The majority of city drivers are commuting from within the city limits.
This is the credited response. If most commuters live within city limits, they will have access to the railway system.
E
A railway system once existed in the city, but was shut down for lack of use.
This is not the the credited choice because it undermines the mayor's plan. If an earlier system wasn't utilized, it's more than likely
that the proposed system will also be underused.
stay away from absolute answers alawys and choose a normal one quiclkly
Argument: Because caffeine, if drunk as coffee, gets partially used up by digestion instead of reaching brain cells, it is less effective
as a stimulant to alertness. Some non-caffeine stimulants can be inhaled, avoiding this problem.
What scientific advance would make caffeine more beneficial as a stimulant to users? Ideally, users want a more effective delivery
than drinking, so look for a choice
Caffeine, such as that in coffee, must still be administered by the relatively cumbersome process of brewing and drinking. If caffeine
is drunk, some of the caffeine gets partially digested and cannot reach the brain to cause alertness. However, certain inhaled non-
caffeine stimulants contain magnetized elements, which are not subject to separation through digestion.
Question
The statements above most strongly support a claim that a research procedure that successfully accomplishes which of the
following would be beneficial to users of caffeine as a stimulant?
Choices
A
Turning coffee into a gaseous cloud of magnetized elements that is absorbed directly by brain cells.
This argument makes the connection between the likelihood of driving drunk and the kind of car one drives. The
evidence states two facts: a small minority of Lalaland highway drivers drove sports cars, while a quarter of all
vehicles whose drivers were ticketed for drunk driving were sports cars. The conclusion is that sports car drivers on
Lalaland highways were more likely to drive drunk than are the drivers of other kinds of vehicles.
The necessary assumption, upon which the conclusion depends, is that those who
are ticketed drive drunk more often than those who are not ticketed.
Hi mneeti
B)Many of the vehicles ticketed for drunk driving were ticketed more than once during the time period covered by the report.
B is wrong because B only mentions that those who were ticketed for drunk driving will be ticketed again during the time period. But what if those
who were ticketed more than one time are are drivers of other kinds of vehicles, NOT drivers of sport vehicles. If thats the case, you cannot
conclude drivers of sport vehicles drunk more regularly than drivers of other kind of vehicles do.
Hence, B is not the assumption.
C)Drivers who are ticketed for drunk driving are more likely to drive drunk regularly than are drivers who are not ticketed.
C means those who are ticketed fro drunk driving are more likely ticketed again. It does not matter drivers of sport cars or other kind of cars. This
is the base for a comparison between the number of ticket/driver of sport cars AND the number of ticket/driver of other kind of cars.
Name drivers of sport cars is SET 1, name drivers of other kind of cars is SET 2.
So, if the number of ticket/driver of SET 1 is greater than that of SET 2, the conclusion is correct.
Ticket / Driver:
20 drivers of sports cars got 25 tickets ==> 1.25 ticket / driver
980 drivers of other kind of cars got 75 tickets ==> 0.07 ticket / driver
Clearly, drivers of sport cars are more likely are more likely to drive drunk regularly than are drivers. Hence, C is the assumption.
Hope it helpsthis assumption question relies on the connection between 25 % and more likely. Assuming that if a higher percentage of people have
something happen to them then that makes it more likely. The answer choices are designed to distract the reader with other, irrelevant issues.
Stiv wrote:
Although only 2 percent of drivers on Lalalands highways drove sports cars, 25 percent of all vehicles ticketed for drunk driving in the past 90 days
were sports cars. Clearly, sports car drivers on Lalaland highways are more likely to drive drunk than are drivers of other kinds of vehicles.
A)Drivers on Lalaland highways drive drunk more often than do drivers on highways not covered in the report. the subject of other highways
B)Many of the vehicles ticketed for drunk driving were ticketed more than once during the time period covered by the report. Being ticketed
more than once would acutally weaken the argument becuase it would show a problem with the statistics
C)Drivers who are ticketed for drunk driving are more likely to drive drunk regularly than are drivers who are not ticketed.this is correct because
D)The number of drivers ticketed for drunk driving was greater than the number of sportscars. This is irrelevant becuase the ratio of
tickets to sportscars doesn't matter unless all of the sportscars were getting tickets
E)Drivers of sports cars are less likely to be ticketed for drunk driving than are drivers of other kinds of cars.While this may strengthen the
argument, it is not an assumption becuase it is a new fact, not a connection of two facts in the argument itself.
This Complete the Passage question is also a Weaken the Argument question.
The marketing team's argument here is that Haplosoft needs better tutorial material because users depend on it to use the software
well. One engineer disagrees, though, and the credited answer will support this point of view. Look for the choice that will disprove
some part of that original claim, whether the need for new materials or the users' dependence on tutorials in the first place.
A recent tracking survey has shown that the vast majority of users of Haplosoft's new text-editing software ignored the
recommended tutorial that comes with the software. Some representatives of Haplosoft's marketing team expressed concern that
the company would need to develop more attention-grabbing tutorial materials, or else users would fail to know how best to use the
software. However, their concerns were assuaged when a Haplosoft engineer pointed out that ___________________.
Question
Which of the following best completes the passage?
Choices
A
most users look to the software to accomplish a limited range of tasks because that is all they know how to do.
This is not the credited choice. This explanation is almost comforting: it would be if the users only needed to do a small range of
tasks, but that's not what this says. This would lend support to the marketing team's worries.
B
the software packaging contains an instruction manual, which has most of the same content as the tutorial.
This is not particularly persuasive: if the users ignore the tutorial, there is nothing that suggests they will be any more likely to read
a manual. This is not the credited choice.
C
the tutorial is based on a rewritten version of specifications given to the marketing team by Haplosoft's engineers.
This is not the credited choice. It tells us that the tutorial's instructions are good, since they come from the same people who made
the software, but tells us nothing about how the users will use it.
D
the tutorial contains information standard to most text-editing software and with which most users are already familiar.
This is the credited response. If this particular software does not differ too drastically from most other text editors, it is relatively
safe to assume that users will already know how to make the most of the software for their needs, and thus ignore the tutorial
without jeopardizing their experience.
E
historically, very few users have contacted Haplosoft's technical support line with questions about how to use the text editor.
This is not the credited choice. It does not suggest anything about the tutorial; it may mean that customers seek help in other
places, or that they are not using the software for more than a few tasks. In either case, it does not weaken the argument that the
fact that the tutorial is not very helpful is bad for users' ability to use the software well.
Reading through the question stem, the bold words need to be specially focussed on.
A recent tracking survey has shown that the vast majority of users of Haplosoft's new text-editing software ignored the recommended tutorial that
comes with the software. Some representatives of Haplosoft's marketing team expressed concernthat the company would need to develop more
attention-grabbing tutorial materials, or else users would fail to know how best to use the software. However, their concerns were
assuaged when a Haplosoft engineer pointed out that ___________________.
A. most users look to the software to accomplish a limited range of tasks because that is all they know how to do. This statement supports the
marketing team's concern that users would not know how to BEST use the software. Incorrect
B. the software packaging contains an instruction manual, which has most of the same content as the tutorial. There is no guarantee that this
additional material too will not be ignored as the recommended tutorial. Incorrect
C. the tutorial is based on a rewritten version of specifications given to the marketing team by Haplosoft's engineers. Doesnt concern the question
at hand; Incorrect
D. the tutorial contains information standard to most text-editing software and with which most users are already familiar. The underlined part
Also, this statement assumes that if users have questions they will contact support line. Incorrect
Argument structure questions ask you to identify the structural outline of the argument, with the specific content removed. As far as
we're concerned, the content just serves to provide meat for the underlying structure: we're interested only in understanding the
structure. Here, the argument begins by describing an innovation developed in response to a unique challenge. Then evidence is
presented of a new discovery in the archaeological record of a similar innovation in the past, in a different location. This new
evidence is used to argue that the unique challenge that led to this particular innovation must have been shared by the people in
the different location. Sometimes the answer choices use this level of content detail and sometimes they are abstracted even
further; in our case, we could say that the argument's reasoning is based on interpreting new evidence in light of existing known
information.
Because of the venomous spur on its hind foot and its excellent swimming ability, the duck-billed platypus presented unique
challenges for aboriginal Australians intent on trapping it. The Yolngu people of northeastern Australia overcame this challenge by
inventing an innovative aqueous spring-loaded trap that could catch platypuses in the water without getting too close to them.
Recent thousand-year-old archaeological discoveries show that the Maori people of New Zealand had in the past used a very similar
aqueous spring-loaded trap. Although today the duck-billed platypus is found only on the coast of eastern Australia and Tasmania,
this discovery shows that in the past duck-billed platypuses were also found on neighboring New Zealand.
Question
Which one of the following most accurately describes the method of reasoning employed in this argument?
Choices
A
Demonstrating how challenges to people's survival lead to innovative solutions
First of all, this ignores the second half of the argument and doesn't describe its final conclusion. In addition, it mischaracterizes the
nature of the venomous spur: we have no reason to think platypus venom posed a challenge to aboriginal people's survival, only
that it challenged their attempts to capture the platypus.
B
Interpreting new evidence in light of existing known information
C
Revising the established theory in light of newly discovered evidence
The established theory, in this case, is how this particular trap was invented in order to trap the duck-billed platypus. But the new
evidence doesn't challenge this theory; it applies it to work toward the conclusion that the platypus was previously
found in areas where it is not found today. Where platypuses are to be found today is not a theory but an observed fact.
D
Dismissing the reliability of new evidence in light of known facts
The argument doesn't dismiss the new evidence--that of the trap remains found on New Zealand--in light of the known facts about
where platypuses can be found today. To the contrary, it accepts these new facts at face value and uses them to develop a theory
that the known facts don't give the complete picture: that even though platypus are not found in New Zealand today, in the past
they were.
E
Explaining the way two groups of people have influenced each other
Although this argument does speak about two peoples, the Yolngu and the ancient Maori, it does not say anything about how they
may have influenced each other. It describes a similarity--they both used similar aqueous spring-loaded traps--but suggests that
similar needs (to trap platypuses) led to the similar innovation, rather than any direct contact between the peoples.
This argument depends on the assumption that the key to winning back clients who have taken their business to Pensacola's rival is
through customer support. In the answer choices below, avoid those that depart from this particular element of the business, and
pick the one that suggests that increasing customer support personnel is ill-advised or unlikely to work.
Pensacola Mail Fulfillment Services found, in a recent survey of its national client base, that some 20% of its customers were less
than completely satisfied with the quality of its service. Pensacola's plan to increase its customer support personnel by one third in
order to regain clients lost to its rival, Good For What Mails You ( LEAVE THIS USELESS CHUNK ) , is likely to succeed unless it is the
case that ___________________.
Question
Which of the following best completes the passage?
Choices
A
Pensacola typically offers a bundled set of services, while Good For What Mails You allows its clients to choose from a comparatively
" la carte" list of options.
This is not the credited choice. It is one reason clients might be tempted to leave Pensacola for its rival, but it has no bearing on
whether or not the plan to regain customers by increasing customer support personnel will work.
B
Pensacola still has many loyal customers of long standing who have expressed no dissatisfaction with their customer service
experience.
This is not the credited choice. It does not affect the argument at hand, which is focused only on how to regain the business of those
clients who are less than fully satisfied.
C
the percentage of Pensacola clients who responded to the survey was approximated to be somewhere between 60 and 70 percent.
This is not the credited choice. The sample size could be significant as part of a larger argument, but given this information alone it
is not sufficient to impact the argument presented here; anyway, this high a rate of response is enough to assume that the non-
responding clients break down in a similar proportion.
D
Pensacola's prices have traditionally been lower than those of Good For What Mails You.
This is not the credited choice. Price does not affect customer service, which is the only variable in the argument proposed here.
E
many of Pensacola's customers objected to the poor training and relative
inexperience of the company's support personnel.
THIS A VERY IMPORATNT STRNETHEN QUESTION ......U NEED TO UNDERSTAND ....THAT CAUSE
---EFFECT CAN BE STRENTHENED BY SYAING NO CAUSE ---- NO EEFECT ...AN EXCELLENT
EXAMPLS OF THAT .........STUDIES PHENOMENON...OBSERVATIOSN BASICALLY GIVE A A CAUSE
AND EFFECT
This argument concerns why product display can affect consumer purchases. Psychologists hypothesize that consumers believe
items displayed on an aisle end-cap are on sale, whether or not the items actually are on sale, and consumers like to buy discounted
items.
You are looking for additional information that will add support to this argument. The credited choice will clarify the link between
these two things without introducing any unnecessary new focus.
Studies in grocery stores show that shoppers purchase more of an item if it is on an aisle end-cap. Psychologists hypothesize that
seeing an item prominently displayed at the end of an aisle makes many shoppers willing to buy more of it because they think the
prominent display means it is on sale, and that they should purchase the item while it is discounted.
Question
Which of the following, if true, most strongly supports the psychologists interpretation of the studies?
Choices
A
The effect noted in the study is also seen in bookstores and large chain discount stores.
This is not the credited choice. That the phenomenon is widespread still doesnt account for why shoppers engage in this kind of
behavior.
B
Shoppers who first see that same item in its regular placement on a shelf are less likely to purchase the item than they are to
purchase comparable, less expensive items.
This is the credited response. This clearly states that the way the product is displayed has an effect on consumer activity. When they
see the same items are in their usual place, customers are more likely to buy less expensive alternatives.
C
Virtually all of the shoppers who purchased items from end-caps used coupons for other items they purchased.
This is not the credited choice. Coupons are out of scope. Just because customers used coupons for other items doesnt mean they
werent motivated by product placement.
D
Sales of any item in a grocery store will increase if it is placed on an end-cap.
This is not the credited choice. Like answer A, the fact that the phenomenon is widespread doesnt explain the shoppers' behavior.
E
A. Shoppers who purchased items picked up from end-caps had lower grocery bills than those who did not.
This is not the credited choice. The overall total of a grocery bill is out of the scope of this argument.
my take wud be B for only B only resonates with the argument .its a simple cause and effect
logic so all B is doing is is that its telling us that on removing the cause the effect is also
vanishing and hence supporting the argument
ONCE U GET WHAT U PRETHOUGHT U R DONE ......
This is an argument structure/role played question. We are asked to identify the underlying structure of the argument stripped of the
content that gives it its particular contours. The first sentence begins with evidence (the CEO ate the spinach) leading to a particular
conclusion (the company's spinach is safe). Then the logic for the conclusion is given. Finally, the conclusion is called into question
(maybe consumers shouldn't trust that the spinach is safe), and a claim is introduced that challenges the conclusion (CEOs in fact
are willing to take physical risks for the good of their company).
When the CEO of Green Farms appeared on television, eating a spinach salad grown by his company, consumers were
convinced that, as the company had maintained all along, the contamination of one batch of the company's spinach with the
virulent E. Coli bacteria had been an isolated incident, and that the rest of the spinach was safe to eat. They reasoned that he
wouldn't put his life at risk, even if the chance of a particular leaf of spinach being infected were small, and that if he was willing to
eat the spinach it must really be safe. The consumers may have been overly trusting, however, sinceCEOs have been known to
take relatively small risks to their physical well-being in order to promote the financial health of their companies.
Question
In the argument given, the two boldfaced selections play which of the following roles?
Choices
A
The first offers evidence that is taken to support a conclusion; the second provides a reason for
questioning that conclusion.
D
The first describes the background circumstances for the argument as a whole; the second
provides an explanation that the argument attempts to prove.
This is not the credited choice. The background circumstances are described in a non-
boldfaced section of the argument.
E
Both offer evidence in support of the conclusion drawn by the argument.
This is not the credited choice. The first selection describes evidence in support of a conclusion called into question by the
argument.
P1:Lark Manufacturing Company initiated a voluntary Quality Circles program for machine operators.
P2: Independent surveys of employee attitudes indicated that the machine operators participating in the program were less satisfied with their work situations after
two years of the program's start.
conclusion: any workers who participate in a Quality Circles program will, as a result, become less satisfied with their job.
we have causal reasoning: participating in quality Circle program --> become less satisfied with their job
All weaken EXCEPT:
A. The second survey occurred during a period of recession when rumors of cutbacks and layoffs at Lark Manufacturing were plentiful weaken.. it
introduces alternative cause for becoming less satisfied
B. The surveys also showed that those Lark machine operators who neither participated in Quality Circles nor knew anyone who did so reported the
same degree of lessened satisfaction with their work situations as did the Lark machine operators who participated in Quality Circles weaken.. it says
becoming less satisfaction is not related to quality circle program
C. While participating in Quality Circles at Lark Manufacturing, machine operators exhibited two of the primary indicators of improved job
satisfaction: increased productivity and decreased absenteeism weaken.. it indicates that the survey data has problem
D. Several workers at Lark Manufacturing who had participated in Quality Circles while employed at other companies reported that while
participating in Quality Circles in previous companies, their work satisfaction has increased the extreme language of the conclusion makes this option a
weakener.. conclusion says any workers who participate in a quality circles program so even if employees participate in another company's quality circle
program and report increase satisfaction, it weakens the argument!!
E. The machine operators who participate in Quality Circles reported that when the program started, the felt that participation
might improve their work situation
this option correctly explains the reason for being less satisfied after participating in quality circle program... their expectation was not
fulfilled.
verSurrender wrote:
In the field of eugenics, geneticists have thoroughly scrutinized the human genome and its ability to shape certain traits to select and pre-
determine an increasing impressive number of physiological characteristics in the unborn, each ready to order to parents
specifications.
B) increasingly impressive number of physiological characteristics in those not yet born, and each one
the word that modifies impressive should be Increasingly. So we can clearly eliminate A and C.
B is awkward- "those not yet born" we would rather prefer simple "unborn" instead of that. So eliminate B.
Same with the D. "all of them" is awkward and redundant. since we have "the unborn"- Noun
B - those not yet born is wordy and we can strike out this option right out
D - all of them - incorrectly modifies "unborn". unborn is a singular and all of them referes to plural form "unborns". Strike out
E - this is correct because - "ready to order to parents specifications - is also a modifer which directly modifies "unborn". We do not need any
additional modifer in this case and this makes E suitable choice.
Best,
Ameya
First published in 1946, and comparing the ways the world appears both in Homers Odyssey and in the Bible, Mimesis, a philosophical treatise
written by Auerbach laid the foundation for a unified theory of western literature beginning from the very early times to his days
(A) and comparing the ways the world appears both in Homers Odyssey and in the Bible, Mimesis, a philosophical treatise written by Auerbach laid
the foundation for a unified theory of western literature beginning from the very early times to his days
(B and laying the foundation for a unified theory of western literature beginning from the very early times to his days, Mimesis, a philosophical
treatise written by Auerbach compared the ways the world appears both in Homers Odyssey and in the Bible.
(C) Mimesis, a philosophical treatise written by Auerbach, compares the ways the world appears both in Homers Odyssey and in the Bible, and lays
the foundation for a unified theory of western literature beginning from the very early times to his days
(D) and comparing the ways the world appears both in Homers Odyssey and in the Bible, Mimesis, a philosophical treatise written by Auerbach, lays
the foundation for a unified theory of western literature beginning from the very early times to his days
(E) and comparing Homers Odyssey with the Bible and the ways the world appears in both , Mimesis, a philosophical treatise written by Auerbach
laid the foundation for a unified theory of western literature beginning from the very early times to his days
(A) and comparing the ways the world appears both in Homers Odyssey and in the Bible, Mimesis, a philosophical treatise written by
Auerbach laid - the foundation for a unified theory of western literature beginning from the very early times to his days
Right choice
(B and laying the foundation for a unified theory of western literature beginning from the very early times to his days, Mimesis, a philosophical
treatise written by Auerbach compared the ways the world appears both in Homers Odyssey and in the Bible.
Wrong because, comparison should come first and then the laying the foundation
(C) Mimesis, a philosophical treatise written by Auerbach, compares the ways the world appears both in Homers Odyssey and in the Bible,
and lays the foundation for a unified theory of western literature beginning from the very early times to his days
wrong
(D) and comparing the ways the world appears both in Homers Odyssey and in the Bible, Mimesis, a philosophical treatise written by
Auerbach, lays the foundation for a unified theory of western literature beginning from the very early times to his days
wrong
(E) and comparing Homers Odyssey with the Bible and the ways the world appears in both, Mimesis, a philosophical treatise written by Auerbach
laid the foundation for a unified theory of western literature beginning from the very early times to his days
E Compares Homers Odyssey with the Bible which alters the intent.
The zoning commission has countered that its consultation process was exhaustive, that it coordinated and participated in over 250 meetings with
leadership councils, business-empowerment boards, and with other community organizations, and that none of the issued permits ought to have
come as a surprise.
A) has countered that its consultation process was exhaustive, that it coordinated and participated in over 250 meetings with leadership councils,
business-empowerment boards, and with other community organizations, and that none of the issued permits ought to have come as a surprise
B) has countered that its consultation process has been exhaustive, that it coordinated and participated in over 250 meetings with leadership
councils, business-empowerment boards, and other community organizations, and that none of the issued permits ought to have come as a surprise
C) has countered that its consultation process has been exhaustive, it coordinated and participated in over 250 meetings with leadership councils,
business-empowerment boards, and other community organizations, and none of the issued permits ought to come as a surprise
D) countered that its consultation process has been exhaustive, that it coordinated and participated in over 250 meetings with leadership councils,
with business-empowerment boards, and other community organizations, and that none of the issued permits ought to have come as surprises
E) countered that its consultation process was exhaustive, that it coordinated and participated in over 250 meetings with leadership councils,
business-empowerment boards, and other community organizations, and none of the issued permits ought to have come as a surprise
B) has countered that its consultation process has been exhaustive, that it coordinated and participated in over 250 meetings with leadership
councils, business-empowerment boards, and other community organizations, and that none of the issued permits ought to have come as a surprise
No obvious problem. Looks good.
C) has countered that its consultation process has been exhaustive, it coordinated and participated in over 250 meetings with leadership councils,
business-empowerment boards, and other community organizations, and none of the issued permits ought to come as a surprise
Leaving out the parallel that's is problematic, because it's unclear whether, for example, "it coordinated and participated in ..." is suppose to be
something else the zoning commission claimed or an independent parallel fact. Ambiguous = wrong.
D) countered that its consultation process has been exhaustive, that it coordinated and participated in over 250 meetings with leadership councils,
with business-empowerment boards, and other community organizations, and that none of the issued permits ought to have come as surprises
Sneaky mistake. Almost everything here is good, but "none" is singular, so it doesn't match "surprises." This is wrong.
E) countered that its consultation process was exhaustive, that it coordinated and participated in over 250 meetings with leadership councils,
business-empowerment boards, and other community organizations, and [???] none of the issued permits ought to have come as a surprise
Here, we have a missing "that", a variant on the inside/outside mistake in (A). We need a "that" in front of each of the three branches of the
parallelism. This is wrong.
Mike
Examples of "tulipomania," a term coined from the tulip craze of the seventeenth-century in the Netherlands, include speculative bubbles in South
Seas trading rights in the 1720s, Victorian real estate in the 1880s, the U.S. stock market in the 1920s, and the obsession for Beanie Babies in the
1990s.
A. Examples of "tulipomania," a term coined from the tulip craze of the seventeenth-century in the Netherlands, include speculative bubbles in
South Seas trading rights in the 1720s, Victorian real estate in the 1880s, the U.S. stock market in the 1920s, and
[Hold it]
bubbles in
B. Examples of "tulipomania," a term coined from the seventeenth-century tulip craze in the Netherlands, include speculative
South Seas trading rights in the 1720s, Victorian real estate in the 1880s, and the U.S. stock market in the
1920s, as well as
[and + as well as = redundancy]
C. Coined from the seventeenth-century tulip craze in the Netherlands, examples of "tulipomania" include speculative bubbles in South Seas trading
rights in the 1720s, Victorian real estate in the 1880s, and the U.S. stock market in the 1920s, as well as
[Misplaced Modifier - Coined ---> examples of tulipomania ]
D. Coined from the seventeenth-century tulip craze in the Netherlands, "tulipomania" includes examples such as speculative bubbles in South Seas
trading rights in the 1720s, Victorian real estate in the 1880s, the U.S. stock market in the 1920s, and
[Misplaced Modifier - Coined --->tulipomania itself ???]
E. "Tulipomania," coined from the seventeenth-century tulip craze in the Netherlands, included speculative bubbles in South Seas trading rights in
the 1720s, Victorian real estate in the 1880s, the U.S. stock market in the 1920s, and
[Misplaced Modifier - Coined --->tulipomania itself ???]
OA indeed is B .
Sentence : Examples of "tulipomania," a term coined from the tulip craze of the seventeenth-century in the Netherlands,include speculative
bubbles in South Seas trading rights in the 1720s, Victorian real estate in the 1880s, the U.S. stock market in the 1920s, and the obsession for
Beanie Babies in the 1990s.
A. Examples of "tulipomania," a term coined from the tulip craze of the seventeenth-century in the Netherlands, includespeculative bubbles in -
*South Seas trading rights in the 1720s,
*Victorian real estate in the 1880s,
*the U.S. stock market in the 1920s, and
? obsession for Beanie Babies in the 1990s. (speculative bubbles in obsession for beanie ...? nonsensical)
So we have a 4 item list for speculative bubbles that doesn't make sense.
B. Examples of "tulipomania," a term coined from the seventeenth-century tulip craze in the Netherlands, include speculative bubbles in
HTH
WHENEVER U SEE A BREAK IN INLIST ...THINK OF ENDING IT RGHT THERE WITH A AN AND TRY A
NEW LIST STRAT...IN SOME OTHER OPTION WITH SOMEO THER PART OF PEECH ...NOUN , NOUN AND
NOUN AND PHRASE AND PHRASE ....if u r not clear with time accurennce of teh sentence alsways
concentrate ...on the tenses used in oginial sentence if they are ...in "ing form" its ok to use it in
parallelsim
in long senetnecs before getting into close nitty grittie grammar ...etc ...jsut see th
structure ....and u wiill see a connect for the ideas ........for seeing the structure ....cut
through all the phrasee....with prepositions ...etc ...just concentrate on the subect verbs
and the meaning ...as what is the sentence trying to say ....just catch hold of what is the
sentenec trying to say
The paradox of the Roman empires history is marked by the fall of the western part around the 5th centaury A.D with several regions succumbing
to cross border terrorism and internal insurgency, breaking into independent kingdoms, the rise of the eastern part, with the empire fire-walling
the crises, and retaking most of the lost territories to establish a firm rule of over most of the Eastern Europe for another six to seven centuries
(A) with several regions succumbing to cross border terrorism and internal insurgency, breaking into independent kingdoms, the rise of the eastern
part, with the empire fire - walling the crises, and retaking
(B) as several regions succumbed to cross border terrorism and internal insurgency and broke into independent kingdoms, and the rising of the
eastern part as the empire fire-walled the crises and retook
C) when several regions succumbed to cross border terrorism and internal insurgency and broke into independent kingdoms, and by the rise of the
eastern part, where the empire fire-walled the crises to retake
(D) with several regions succumbing to cross border terrorism and internal insurgency and breaking into independent kingdoms, and by the rise of
the eastern part, with the empire fire -walling the crises, and retaking
(D) The paradox of the Roman empires history is marked by the fall with several regions succumbing to cross border terrorism and internal
insurgency and breaking into independent kingdoms, and The paradox of the Roman empires history is marked by the rise of the eastern part, with
the empire fire -walling the crises, and retaking
(E) amid several regions succumbing to cross border terrorism and internal insurgency broke into independent kingdoms, and by the rise of the
eastern part, as the empire fire -walling the crises, retook
(A) with several regions succumbing to cross border terrorism and internal insurgency, breaking into independent kingdoms, the rise of the eastern
part, with the empire fire - walling the crises, and retaking
Unparallel; no - by - before the rise
(B) as several regions succumbed to cross border terrorism and internal insurgency and broke into independent kingdoms, and the rising of the
eastern part as the empire fire-walled the crises and retook
Unparallel; no - by - before the rise
C) when several regions succumbed to cross border terrorism and internal insurgency and broke into independent kingdoms, and by the rise of the
eastern part, where the empire fire-walled the crises to retake
-several regions succumbed and broke- is not parallel to - empire fire-walled the crises to retake The first part is using two simple past
tenses namely succumbed and broke while the latter is using a simple past and an infinitive fire-walled and to retake. IMO, this error
downgrades C
The prepositional phrase - with several regions succumbing to cross border terrorism and internal insurgency and breaking into independent
kingdoms is perfectly matched by an equally parallel - with the empire fire -walling the crises, and retaking - both parts using present
participles with vantag
This is why this is the OA.
(E) Amid several regions succumbing to cross border terrorism and internal insurgency broke into independent kingdoms, and by the rise of the
eastern part, as the empire fire -walling the crises, retook
Structurally flawed and fragmented without conjunction
Thanks to those brave hearts who have given us great insight into their thinking and I regard those opinions
JUST GO NIT PICKING TRY TO MATCH WVVERY SUBJECT VERB ...EVERY PRONOUN REFERNCE EVERY
PARALLELSIM ..EVERY TENSE ...EVEVRY MODIFIER IN THE ORIGINAL SENTNECE AND ...GET THE
MEANING ...ALONG WITH THE STRUCTIRE ...N U WILL NAIL IT
never cross out a not so appealing sentence jsut put a side cross...and not on it
thats means style mistake ......and go on nitpicking ....if alll are having major
grammatical errors then ....that stylistically not so appealing will be ur answer
else the other one will be ur asnwer
(B) use of the term turnover and the term revenue in business across the Atlantic; in the United States, turnover refers to how rapidly the inventory
acquisition and transaction occurs whereas
OCCUR must be used instead of OCCURS coz the subject " the inventory acquisition and transaction ( X and Y)" is plural. Thus Incorrect
(C) way the term turnover and the term revenue are used in business across the Atlantic; in the United States, turnover refers to how rapidly the
inventory acquisition and transaction occur since
- Author want to make comparison rather than to give Reason why something happened. Thus Incorrect
(D) usage of the term turnover and the term revenue in business across the Atlantic; the rapidness of inventory acquisition and transaction is called
turnover in the United States whereas - CORRECT
(E) way the term turnover and the term revenue are used in business across the Atlantic; in the United States, the rate at which the inventory
acquisition and transaction occur are called turnover whereas
Singular verb (IS) must be used instead of ARE coz the subject "RATE" is singular. Thus Incorrect
blueseas wrote:
RAPIDLY ==>This is an adverb
Hi Blueseas,
RAPIDLY is not modifying any Noun in option B. It is correctly modifying VERB OCCURS (RAPIDLY..... OCCURS)
WHEN ALL LOOK INCORRECT SEE THE FIRST ONE REAALY CAREFULLY AS IN THE BEGINNING
U DO NOT THE INETENDED MEANING ....AND THUS AS TIMES DO NITPICKING TO
ELIMINATELY USELESSLY .....ALSO DO NOT ELIMINATE OPTION A AT FISRT GO UNLESS A
BLATANT LY WORNG ANSWER IS TEHERE
avohden wrote:
Before converting to Christianity and marrying Jadwiga, the second of Poland's Angevin
rulers, thereby becoming King of Poland, Wladyslaw II was known as Jogaila, Grand
Duke of Lithuania.
(A) Before converting to Christianity and marrying Jadwiga, the second of Poland's
(B) Before he converted to Christianity, married Jadwiga, the second ruler of Poland's
Angevin, and became King of Poland, Wladyslaw II was known as Jogaila, Grand
Duke of Lithuania.
(C) Before marrying Jadwiga, the second of Poland's Angevin rulers, thereby converting
to Christianity and becoming King of Poland, Wladyslaw II was known as Grand Duke
of Lithuania, Jogaila.
(D) Before Wladyslaw II converted to Christianity and married Jadwiga, the second ruler of
Poland's Angevin, thereby becoming King of Poland, he was Jogaila, Grand Duke of Lithuania.
(E) Before Wladyslaw II converted to Christianity and married Jadwiga, the second of Poland's
Angevin rulers, thereby becoming King of Poland, he was Grand Duke of Lithuania, Jogaila.
Interesting question - Has so many modifiers and commas, none of the answers seem correct.
Honestly, I used the process of elimination here. I read A and jumped into the other choices (Not the best way to do..w/o prethinking)
Though we definitely have a meaning issue here beside a modifier issue.
What the org sentence is trying to say is: Wladyslaw II became the king of Poland after marrying Jadwiga( second of Poland's Angevin rulers). But
before converting to Christianity and marrying Jadwiga, he was known as Jogaila (Grand Duke of Lithuania)
C - Before marrying Jadwiga, the second of Poland's Angevin rulers, thereby converting to Christianity and becoming King of Poland, Wladyslaw II
was known as Grand Duke of Lithuania, Jogaila.
This sounds like he became the king before he married Jadwiga and he converted to Christianity because of that.
D- Before Wladyslaw II converted to Christianity and married Jadwiga, the second ruler of
Poland's Angevin, thereby becoming King of Poland, [color=#0000ff]he was Jogaila, Grand Duke of Lithuania.[/color]
'he was Jogalia' as a stand alone phrase seems incorrect. It sounds like the second ruler of Poland's Angevin became King of Poland.
thereby becoming King of Poland incorrectly modifies the second ruler of Poland's Angevin
E- Before Wladyslaw II converted to Christianity and married Jadwiga, the second of Poland's
Angevin rulers, thereby becoming King of Poland, he was Grand Duke of Lithuania, Jogaila.
This almost seems correct, but the statement ends in Jogalia! It's like saying: He was smart, Tom
P.S: IF the words are too confusing, you can shorten them. I translated Jadwiga and Jogalia to Jad and Jog so I could read faster!
ALWAYS WRITE A KEYWORD IN LARGE OPTION QUESTION FOR ELIMINATING AN
OPTION ...........IT WILL BE USEFUL IN FINAL SELECTION HCOICE .......HERE I KNEW USED
OFTEN IS WRONG STILL I FORGOT IT LATER AND CHOSE A AGAIN AS NOTHING WAS
SUITING ...BEWARE
The word inspired is used often by numerous critics in their description of a great play; it is precisely the emotion experienced by the audience as
the final curtain descends.
A. The word inspired is used often by numerous critics in their description of a great play; it is precisely the emotion experienced by the audience
as the final curtain descends
B. Used often by numerous critics in their description of a great play is the word inspired, which is the precise emotion experienced by the
audience as the final curtain descend
C. Precisely describing the emotion of the audience as the final curtain descends is the word inspired, often used by
numerous critics to describe a great play
D. The word inspired is often used by numerous critics in their descriptions of a great play and describes the emotion of the audience as the final
curtain descends as well.
E. The word inspired is the precise emotion of the audience as the final curtain descends and is used by numerous critics to describe a great play.
'Used often' is unidiomatic -> should be 'often used' instead. So options A and B are out.
C/E : The word X................IS Used to describe a play ( Guess this sounds much better / concise )
Bxn C & E :
E : The word inspired is the.............. emotion of the audience ( The word cannot be an Emotion............ It can describe / symbolise an
Emotion ) : Eliminated
C : Describing the emotion of the audience ...................is the word inspired ( The word Inspired....Describes/ Symbolises the emotion of
audience ) : Perfect
Leading to C, my take.
A 2009 study from the California State Housing Authority concluded that conversion from ownership to rental properties has often been difficult; it
has been more common for some townhouses and other attached homes that are relatively small and old, located in central cities.
(A) difficult; it has been more common for some townhouses and other attached homes that are relatively small and old, located in central
cities.
(B) difficult; it has been more common for some townhouses and other attached homes that are relatively small, old, and that are located in
central cities.
(C) difficult; it has been more common for some townhouses and other attached homes, which are relatively small and old, and located in central
cities.
(D) difficult: It has been more common for some townhouses and other attached homes that are relatively small and old
and located in central cities.
(E) difficult: It has been more common for some townhouses and other attached homes that are relatively small and old, and located in central
cities.
[Reveal] Spoiler: OA
Hi there,
In a parallel list, comma is used before the parallel marker or the connector that joins all the entities in the list when there are more than two
entities in a list. For example:
Sri love apples and mangoes.
Sri loves apples, mangoes, and grapes.
Since there are only two entities in the first sentence, we don't need comma before "and". But since there are three entities in the list in the
second sentence, we need comma before "and".
In the given question, there are two lists. The second list resides in the first list. However, both the lists have only
two entities.
Entities in the main list - "are relatively small and old" (are) located in central cities.
Entities in the sub-list - small and old.
This means that we don't need comma before any of the "and" because they join only two entities in the list. You can study other official
sentences for this usage of comma.
Hi,
'Which' vs 'that' is a core SC concept that you need to learn properly. Any decent SC book will do this.
For example:
Galileo's 4th theory (there is only 1 4th theory) which (no need to define) talks about xyx
Galileos' theory (many theorys) that (need to define) talks about xyz[/i}
James
Hi,
==>The colon (:) provides further explanation for what comes before it. For example, you can
use a colon to equate a list with its components. You should be able to insert the word
namely or the phrase that is after the colon.
What comes before the colon must be able to stand alone as a sentence. What comes after
the colon does not have to be able to stand alone.
You can put a main clause after a colon as well. The key is that this clause must explain
what precedes the colon-perhaps the entire preceding clause.
example: On January 1, 2000, the national mood was completely different from
what it would become just a few years later: at the turn of the century,
given a seemingly unstoppable stock market and a seemingly peaceful
world, the country was content.
The words after the colon, at the turn of the century. " was content, can stand alone as a sentence.
They serve to explain the entire clause that comes before the colon (a clause that
asserts an upcoming change in the national mood, as of the first of the year 2000).
Do not confuse the semicolon (;) with the colon (:). The semicolon connects two related
independent clauses, but the second does not necessarily explain the first. In contrast, the
colon always connects a sentence with a further explanation.
hope it helps
d)difficult: It has been more common for some townhouses and other attached homes that are relatively small and oldand located in central
cities.
d)difficult: It has been more common for some townhouses and other attached homes that are relatively small and old located in central cities.
difficult: It has been more common for A that are relatively B and C.
here:
A=SOME TOWNHOUSES AND OTHER ATTACHED HOMES====>SEE THIS IS A COMPOND SUBJECT AND HENCE PLURAL ...AND THIS IS WHYARE IS USED
AFTER THAT.
B=SMALL AND OLD
C= LOCATED IN CENTRAL CITIES
B and C are the characterstics of A.
OLD LOCATED IN CITIES doesnt makes sense....
hope it helps.
t is no surprise that Riyadh, the Saudi capital where people revere birds of prey and ride camels regularly, is home to the largest hospital for
falcons, a place where falcons from all over the world are treated in operating rooms, an ophthalmology department, and a pox area, and to the
largest veterinary clinic for desert mammals, a place where camels and other desert species are expertly cared for.
D to an ophthalmology department, and to a pox area and the largest veterinary clinic for desert mammals, a place where camels and other
desert species are expertly cared for.
This sentence uses parallelism to describe the hospital and not to put on the same level the hospital and the clinic: wrong structure.
A an ophthalmology department, and a pox area, and to the largest veterinary clinic for desert mammals, a place where camels and other desert
species are expertly cared for.
B an ophthalmology department, to a pox area, and to the largest veterinary clinic for desert mammals, a place where camels and other desert
species are expertly cared for.
B says that the city is home to the largest hospital, to a pox area, and to the largest veterinary clinic. By using "to a pox area", B places it at the
same level as the hospital and the clinic, so the clause "a pox area" is no more part of description of the hospital itself.
Use parallelism to put on the same level things that express a similar concept.( a parallel idea)
Example:
1)I lived in Miami, a city in Florida, and in Boston. From this sentence you get that I lived in Miami and in Boston.
2)I lived in Miami, in Florida and in Boston. From this sentence you get that I lived in Miami, in Boston AND in Florida(unspecified city).
Among the important themes that characterize Chinese history are the patterns of dynastic rise and fall, intermittent aggression from northern
aliens, and varying degrees of openness to outside cultural influences and the dynamics of stability and social harmony
A. Among the important themes that characterize Chinese history are the patterns of dynastic rise and fall, intermittent aggression from northern
aliens, and varying degrees of openness to outside cultural influences and the dynamics of stability and social harmony
Incorrect, P,Q,R & S cannot be parallel simultaneously as they belong to different clause category.
B. Among the important themes that characterize Chinese history are the patterns of dynastic rise
and fall, and intermittent aggression from northern aliens, together with varying degrees of
openness to outside cultural influences and the dynamics of stability and social harmony
Correct. P is parallel to Q, [connector] R is parallel to S
C. The important themes that characterize Chinese history are the patterns of dynastic rise and fall, intermittent aggression from northern aliens,
along with varying degrees of openness to outside cultural influences and the dynamics of stability and social harmony
Incorrect. Here Q follows P but is not separated by parallel marker "and". R is parallel to S.
D. Among the important themes of Chinese history are the patterns of dynastic rise and fall, intermittent aggression from northern aliens, varying
degrees of openness to outside cultural influences and the dynamics of stability and social harmony that characterize its history
Incorrect. Same as A. Also the phrase "the dynamics of stability and social harmony that characterize its history" changes the intended meaning of
"themes that characterize its history"
E. The important theme that characterizes Chinese history is the pattern of dynastic rise and fall, intermittent aggression from northern aliens,
along with varying degrees of openness to outside cultural influences and the dynamics of stability and social harmony
Incorrect. Same as C
Answer: B
Whereas lines of competition are clearly defined in the more established industries, in the Internet industry they are blurred and indistinct, as
companies that compete one day may be partners the next.
A. Whereas lines of competition are clearly defined in the more established industries, in the Internet industry they are blurred and indistinct, as
companies that compete
Correct.
B. Although the lines of competition are clearly defined in industries that are more established, they are blurred and indistinct in the Internet
industry, as competing companies
Wrong.
"the more established industries" differs from "industries that are more established". If you say "THE more + Adjective + Noun" ==> [more
adjective] plays as adjective that modifies the noun directly. But when you say "industries
that are more established ....
(than what?)" ==> you're comparing industries with some thing else. But there's nothing to compare
here --> the comparison is incomplete. (correct grammar is: .... more ....than....)
C. The lines of competition are clearly defined in the more established industries, unlike the Internet where they are blurred and indistinct, as
companies that compete
Wrong. Comparison problem: the lines vs. the internet.
D. Unlike more established industries, where the lines of competition are clearly defined, they are burred and indistinct in the Internet industry, as
companies that compete
Wrong. Pronoun problem. "they" refers to what?
E. Unlike more established industries, with clearly defined lines of competition, those of the Internet industry are blurred and indistinct, as
competing companies[/quote]
Wrong. Pronoun problem. "those" refers to what?
Hope it helps.
_________________
Always do not loose patience and ignore the last option if u have spent 5 mins another 20 seconds is worth it
U mighthave 2 conclusioon or 3 views but always concentrate as to what is the question stem asking may be
quetsio n stem is asking about a particular view and you are giving some other answer a bluner be careful
in any cr fisrt identify what is a fact ...and tus premise and what is opninion and thus a
conclusio...and read what is being asked ....concentrate on scope and chenage or jump in
thoughts while reading the premise and the conclusion
A management protocol system that offers a unique method for the administration of every operational factor from
rules to be obeyed by employees to the scheduling of equipment and supply purchases can be installed by any
hospital that wishes to do so, free of charge. The board of directors at the Amgudnow Hospital, which is
never fully occupied or excessively engaged, perceives the system as being very beneficial financially
and so are planning to utilize it to its fullest. It has been proven that since the system results in a cooperative
work environment with streamlined efficiency, its correct operation leads to a high level of satisfaction in workers and
patients alike, reducing recovery times.
Which of the following, if true, casts the most doubt regarding the effectiveness of the plan of the hospital's
board of directors?
(A) The management system has been applied to many hospitals similar in size to Amgudnow though for most it was
the not the first of such protocols to have been adopted, others usually costing high prices for their use.
(B) Although an effective protocol can be used to completely restructure an inefficient business or organization, the
success of such a project depends heavily upon the willingness of the staff to accept new ideas and try different
approaches to existing problems.
(C)Patients who have experienced professional and compassionate service while receiving care at a hospital are
likely to recommend that hospital to others, and may even be willing to make a donation for the continuation of such
service.
(D) The bankruptcy of a hospital that in the past had suffered from legal problems caused by unprofessional patient
handling as a result of being fully-occupied constantly was prevented due to the system's insight.
(E)Whether covered by medical insurance or paid for privately by an individual, hospital services are extremely
expensive primarily because of the high cost of their trained employment, and are charged for per day of
hospitalization.
[Reveal] Spoiler: OA
E
Weakening Data:
You are required to find data that weakens the conclusion that by
using the system the hospital be financially beneficial. The correct
answer could be one that exposes the argument's assumption.
This answer choice states that hospital services are paid per day.
Since we are told in the last premise that the system results in shorter
recovery times, it follows that patients will be paying lower fees. Since
the Amgudnow is never full, it doesn't need to constantly vacate beds.
Therefore, the system will be too efficient, meaning that the patients
will leave too soon with only small hospital bills and less financial
benefit to the hospital
DO NOT OVERSPEND TIME ON VERY MATHS INTENSVE QUESTION AS U WILL END UP
SCREWING THEM ANYWAYS
track fact and opiniions and then write keyword of scope while reading ....so that u know
where it pinchesthe most
In the modern era, society should accept the premise that equality before the law is not appropriate to the
marketplace, where modern methods of marketing, packaging, and financing have reduced the ordinary consumer to
a subordinate position. To protect the consumer from the hardship and oppression that could result from this inferior
position, the law should limit the freedom of action of the enterprises with which the consumer deals.
Which of the following, if true, would most weaken the argument above?
a)When a consumer buys a product in a cardboard box or can, there is no way to know what there is inside.
b)The concept of equality is contradicted by the reality that the consumer is not equal in power to the supplier
c)The health and well-being of consumers are protected by a variety of statutes that have been in effect for decades.
d)Equality before the law is an essential part of the American way of life and should not be restricted.
e)Society should recognize the right of consumers not to be subjected to unfair and deceptive advertising practices.
ps. Sorry guys forgot to copy paste the question :D and btw to the guys who couldnt deduce what the question was
about even after looking at the "weaken" written in the title need to seriously work on their inference skills
[Reveal] Spoiler: OA
In the modern era, society should accept the premise that equality before the law is not appropriate to the
marketplace, where modern methods of marketing, packaging, and financing have reduced the ordinary consumer to
a subordinate position. To protect the consumer from the hardship and oppression that could result from this inferior
position, the law should limit the freedom of action of the enterprises with which the consumer deals.
the source of this question is evidently from beatthegmat.com practice questions content. A lot of the answers in
their practice question banks are pretty flawed. Bad question. End of story.
Here, the conclusion appears in the last sentence. The author takes a firm stance in declaring that equality before
the law is not appropriate in the marketplace and that the consumer should be protected.
(D) rejects the fundamental premise of the argument that equality before the law is not appropriate to the modern
marketplace.
(A), (B) and (E) is incorrect.
(C) is incorrect.
(C) tells us that consumer protection laws (which may restrict equality before the law) are already in effect, but it
does not contradict the premise of the argument.
the source of this question is evidently from beatthegmat.com practice questions content. A lot of the answers in
their practice question banks are pretty flawed. Bad question. End of story.
Here, the conclusion appears in the last sentence. The author takes a firm stance in declaring that equality before
the law is not appropriate in the marketplace and that the consumer should be protected.
(D) rejects the fundamental premise of the argument that equality before the law is not appropriate to the modern
marketplace.
(C) is incorrect.
(C) tells us that consumer protection laws (which may restrict equality before the law) are already in effect, but it
does not contradict the premise of the argument.
Instead of blaming an airline accident on pilot error, investigators should find out why the error was made by
analyzing airplane design, airline management, and pilot-training programs. For only then can changes be made to
ensure that the same type of error does not recur and cause another accident.
Which of the following is a presupposition of the argument above?
(A) Pilot error is not a contributing factor in most airline accidents.
(B) Airline companies themselves should be the agents who investigate airline accidents.
(C) Stricter government regulation of airline companies will make air travel significantly safer.
(D) Investigators of airline accidents should contribute to the prevention of future accidents.( repeted
..alerweday given )
(E) Most pilots who make errors in flying will repeat their errors unless they are retrained.
I marked "D" but IMO answer choice E is a correct assumption. Premise "Instead of blaming the pilot,
investigators should analyse design, airline management and training programs". Option D is stated in the
argument so it cant be the assumption..In choice E it states " if we only blame the pilots for the accidents always,
then the real cause of the accidents we will never know..AND as the reason is unknown, there is always a possibility
that the same error will occur again. We cant trained the pilots for the unknown errors...we must know the right
reason only then it can be rectified. Only blaming the pilot will not solve the issue as they are always prone to make
the same error.
Public health advocate: It is generally true that medications that undergo the extensive FDA Phase III clinical safety
testing are much safer than less-researched drugs. It is also true that whenever such trials are conducted,
fewer people have experienced unexpected harmful side effects, thus reducing public health risks.
However, eliminating the requirement that even FDA-tested medications continue to include extensive warnings
about individual risk factors would almost certainly harm rather than help public health. Consumers would tend to
rely on the FDAs general certification of safety, and if no longer encouraged to read about individual risks
and drug interactions, many patients would suffer serious adverse reactions.
The two bolded statements serve what purpose in the context of the public health advocates argument?
A The first is a general pattern that the advocate accepts as true; the second is said to be a natural consequence
that must follow if the general pattern applies.
B The first is a causal relationship that the advocate believes will happen again in the case at issue; the second
admits a situation in which the relationship
would not hold.
C The first describes a cause and effect relationship that the advocate believes will not hold in the case at issue;
the second suggests a consideration that supports that belief.
D The first is proof that the advocate uses to support a prediction; the second states that prediction.
E The first acknowledges a consideration that weighs against the stance that the advocate supports; the second is
that stance.
The first is what: for sure not the conclusion so is or a Premise or X (something else) here is a premise in favour of
the conclusion (notice is also true just before).
the second: is nont the conclusion but in somehow support the conclusion though i'm not quite sure if support
completely the conclusion of the argument Consumers would tend to rely on the FDAs general certification
of safety............it seem something in the middle (notice IF) but for sure not against
Ok after this brief brainstorming in my mind I need a: P and a P (both in somehow are in favour of the conclusion
but all things are fuzzy, unclear, I'm not pretty sure -------> this is what I would do during the test)
E The first acknowledges a consideration that weighs against the stance that the advocate supports
based on the first bold AND also on the whole context i do not see anything about a position. Here the advocate
describe something that could happens if something else is not conducted
I do not see here a stance but instead a consideration (notice IF at the beginning of the bold)
Good discussion ..
And the OA: C
Kudos to C
Official explanation:
This is the credited response. Phrase #1 gives a generally true causal relationship (FDA testing produces safer
drugs), but this relationship will not occur in a particular case where another variable is changed (i.e., if testing
results in fewer required warnings). Phrase #2 explains that the usual effects (safer drugs) will not apply in such
circumstances because of a REASON (i.e., because people will stop thinking about individual risks).
Always question whether it is must be true in the last two choices for a sssumption
question ...TRY AND NEGATE THE OPTION TO GET THE TO THE CORRECTASSUMPTIONOR
ANY CR ..JUST COUNTER IT
For the writers who first gave feudalism its name, the existence of feudalism presupposed the existence of a noble
class. Yet there cannot be a noble class, properly speaking, unless both the titles that indicate superior, noble status
and the inheritance of such titles are sanctioned by law. Although feudalism existed in Europe as early as the eighth
century, it was not until the twelfth century, when many feudal institutions were in decline, that the hereditary
transfer of legally recognized titles of nobility first appeared.
The statements above, if true, most strongly support which one of the following claims?
(A) To say that feudalism by definition requires the existence of a nobility is to employ a definition that distorts
history.
(B) Prior to the twelfth century, the institution of European feudalism functioned without the presence of a dominant
class.
(C) The fact that a societal group has a distinct legal status is not in itself sufficient to allow that group to be properly
considered a social class.
(D) The decline of feudalism in Europe was the only cause of the rise of a European nobility.
(E) The prior existence of feudal institutions is a prerequisite for the emergence of a nobility, as defined in the
strictest sense of the term.
this post
I pick E, but I think I understand why A is right now.
Argument:
History: Feudalism is in the same time with noble class, in 8th century.
Nobility, first appear in 12th century.
A - Nobility appear first Feudalism is wrong according to history ==> Like what the argument said
B - Dominant class could be king, knights... we don't know, so we can't say for sure feudaslim functioned without a
dominant class -> WRONG
C - Societal group and social class were not discussed -> Out of scope -> WRONG
D - "The only cause" -> Too extreme, without proper evidence in the argument -> WRONG
E - Feudal institutions is a prerequisite of nobility -> sound like Feudal is a MUST for nobility to happen, there is no
evidence in the argument suggest that -> WRONG
rohitgoel15 wrote:
For the writers who first gave feudalism its name, the existence of feudalism presupposed the existence of a noble
class. Yet there cannot be a noble class, properly speaking, unless both the titles that indicate superior, noble
status and the inheritance of such titles are sanctioned by law. Although feudalism existed in Europe as early as
the eighth century, it was not until the twelfth century, when many feudal institutions were in decline, that the
hereditary transfer of legally recognized titles of nobility first appeared.
The statements above, if true, most strongly support which one of the following claims?
(A) To say that feudalism by definition requires the existence of a nobility is to employ a definition that distorts
history.
(B) Prior to the twelfth century, the institution of European feudalism functioned without the presence of a
dominant class.
(C) The fact that a societal group has a distinct legal status is not in itself sufficient to allow that group to be
properly considered a social class.
(D) The decline of feudalism in Europe was the only cause of the rise of a European nobility.
(E) The prior existence of feudal institutions is a prerequisite for the emergence of a nobility, as defined in the
strictest sense of the term.
First, you must understand what feudalism is. Here is an excerpt from the online dictionary:
Feudalism: A system of obligations that bound lords and their subjects in Europe during much of the Middle Ages. In
theory, the king owned all or most of the land and gave it to his leading nobles in return for their loyalty and military
service. The nobles in turn held land that peasants, including serfs, were allowed to farm in return for the peasants'
labor and a portion of their produce. Under feudalism, people were born with a permanent position in society.
It is the legal and social system that evolved in W Europe in the 8th and 9th centuries, in which vassals were
protected and maintained by their lords, usually through the granting of fiefs, and were required to serve under them
in war...
For writers who coined the term feudalism, existence of noble class was a must. Yet, the author says that legal
sanction of status and inheritance of titles are a must to have a noble class. The author also says that feudalism
existed in 8th century but inheritance of titles got legalized only in 12th century. The issue then is that how can
"feudalism - the way it is defined" exist in 8th century if title inheritance was not legal at that time. So as far as
actual history is concerned, existence of a nobility (status and title inheritance) cannot be necessary
(A) To say that feudalism by definition requires the existence of a nobility is to employ a definition that distorts
history.
As for (C),
(C) The fact that a societal group has a distinct legal status is not in itself sufficient to allow that group to be properly
considered a social class.
This cannot be inferred from the argument. The argument tells us that legal status is necessary to
consider a societal group a social class. Whether it is sufficient or not, we do not know. Also, the
argument only talks about the noble class; we don't know whether it holds for all social classes.
NEVER ELIMINATE THE FIRST OPTION RIGHT AWAY ...BACAUSE U HAVE JUST STARTED AND DO NOT
KNOW IF THE OTHER OPTION SHAVE BIGGER FALLACY ..IF U FEEL IT IS INCORRECT JUST PUT A SIDE
CROSS .....WITH A KEYWORD WHY
Last January, in an attempt to lower the number of traffic fatalities, the state legislature passed its
Click It or Ticket law.
Ok this is a fact
Under the new law, motorists can be pulled over and ticketed for not wearing their seat belts, even if
an additional driving infraction has not been committed.
it seems that the new law going to be most sever than before. also notice the meaning of this sentence, basically it
says that the drivers could be ticketed even if they do not have previous infractions
Lawyers and citizens groups are already protesting the law, saying it unfairly infringes on the rights of
the states drivers.
Law enforcement groups counter these claims by stating that the new regulations will save countless
additional lives.
The new law will save additional lives. This could say that the old law save lives but lower than the new law, but it
still saves lives
(A)Prior to the Click It or Ticket law, motorists could not be stopped simply for not wearing a seat
belt.
This seems suddenly good because with the new law the drivers could be stopped for not wearing a seat belt even if
they do not commit any infraction. Instead with the previous law they could be ticketed for bealt only in combination
with another infraction
(B)The Click It or Ticket law violates current search and seizure laws.
I do not see anything in the stimulus that talks about seizure of something (I do not care about THAT something
even if I know or not
(C)Laws similar to Click It or Ticket have effectively reduced traffic fatalities in a number of states.
I do not see anything about traffic fatalities maybe I can infer that if I do not wear the seat blet something could
happen to me or my family but this is to far from "home" i.e. the stimulus
This one is tricky. Because the previous law was ineffective in saving lives AT ALL. But we know from the last
sentence that this is not true completely
(E)Law enforcement groups, rather than citizens groups, should determine how to best ensure the
safety of motorists.
A must be the answer and you infer it from the second phrase of the stimulus
regards
ALWAYS READ THE ENTIRE OPTION IN THE START THINGS MAY LOOK OPPOSITE
BUT THEY ACTUALLY ARE NOT BE CAREFUL ABOUT IT
Hardin argued that grazing land held in common(that is, open to any user) would always be used less
carefully than private grazing land. Each rancher would be tempted to overuse common land because
the benefits would accrue to the individuals, while the costs of reduced land quality that results from
overuse would be spread among all users. But a study comparing 217 million acres of common grazing
land with 433 million acres of private grazing land showed that the common land was in better
condition.
Which of the following, if true and known by the ranchers, would best help explain the results of the
study?
(A): With private grazing land, both the costs and the benefits of overuse fall to the individual user.
(B): The cost in reduced land quality that is attributable to any individual user is less easily measured with common
land than it is with private land.
(C): An individual who overuses common grazing land might be able to achieve higher returns than other uses can,
with the result that he or she would obtain a competitive advantage.
(D): If one user of common land overuses it even slightly, the other users are likely to do so even more,
with the consequence that the costs to each user outweigh the benefits.
(E): There are more acres of grazing hand held privately than there are held in common.
Before I start, try not to post the OA until most people has a shot at it.
Conclusion: Common grazing land is in better condition than private grazing land even though common grazing land
tend to be overused.
Which of the following, if true and known by the ranchers, would best help explain the results of the study?
(A): With private grazing land, both the costs and the benefits of overuse fall to the individual user.
- does not explain why common grazing land is in a better shape, or why private grazing land is in a poorer condition
(B): The cost in reduced land quality that is attributable to any individual user is less easily measured with common
land than it is with private land.
- Again provides us with no useful information why common land is in a better shape
(C): An individual who overuses common grazing land might be able to achieve higher returns than other uses can,
with the result that he or she would obtain a competitive advantage.
- out of scope.
(D): If one user of common land overuses it even slightly, the other users are likely to do so even more, with the
consequence that the costs to each user outweigh the benefits.
- If this was true, common land would be less appealing to ranchers and they would rather use private grazing land
than common grazing land. The consequence would be a lower quality private grazing land.
(E): There are more acres of grazing hand held privately than there are held in common.
- not important. All it tells us is there are more private grazing land than common grazing land.
(D) it is.
the Rotunda Beach city authorities posted signs cautioning against the consumption of the crabs.
Tests revealed that the crabs were high in prestic acid, a dangerous chemical.
But a marked decline in the number of goldfish found by the beach has led to the removal of the warning signs.
Which of the following, if true, would best explain the removal of the signs?
HERE AGAIN TRY TO SEE THE JUMP ....THERE MUST BE A RELATOION BETWEEN PRESTIRC ACID AND
THE FISH ....NOW OPTION A IS OPPOSITE ...JUST CONCENTRATE AND PUT THE REASONING ...IN
SIMPLE WORDS OF THE PREMISES .....NO FISH >>> SAFE >>>>NO PRESTIRC ACID ..............SO
ROTUNDA FISH >>> PRESTORIC ACIDD....SOMETHING LIKE THIS
A. Goldfish found off Rotunda Beach cause hermit crabs to excrete prestic acid.
(If Goldfish causes crabs to excrete acid then their present should reduce prestic acid and cause removal of warning signs.
but instead their absence seemingly resulted in the same. This further exacerbates the paradox adding more confusion.)
B. The amount of prestic acid produced by an individual goldfish has a direct positive correlation with the density of the goldfish
population in that goldfishs immediate vicinity.
(if goldfish population amount of prestic acid
then decline in gold population indicates decline in the amount of prestic acid.
that is the reason why warning signs against eating crabs is removed since the amount of prestic acid reduce considerably.
This explains the paradox)
E. Goldfish would not be able to live in the waters off Rotunda Beach if those waters were not saturated with a precise mixture
of acids, including prestic acid
(This is a double negative.
This means that If those waters are saturated with mixture of acids including prestic acid then Gold fish can live in waters off
Beach.
so if Gold fish is decreasing then those mixture of acids is decreasing in content.
but
mixture of acids is not the same as prestic acid as it is possible that the mixture of acids may not be as harmful as prestic acd.
since we don't have info whether mixture is as harmful or harmless compared to prestic acid, we cannot infer any info which
relates this info to do with crabs infection and removal of warnings.)
always try to reason a choice by reversing the senario see what will be
the response tha....if it is still the same there is no pint using tha
choice as that is redundant choice
More airplane accidents are caused by pilot error than any other single factor. The military recently stopped requiring its pilots to
obtain immunization shots against chemical warfare agents. These shots are known to cause unpredictable dizzy spells which can
result in pilot error. Since many military pilots also pilot commercial passenger airliners, the reason for the military's decision
must have been to reduce the number of commercial airline accidents.
SUPPORT MEANS ....TO ADD CREDENCE TO ONE ARGUMENT ....OR REMOVE CREDENCE
...FROM OTHER
Which of the following, if true, provides most support for the conclusion drawn above?
a. Recently, more pilots have been volunteering for the immunization shots.
b. All commercial airline flights are piloted by two co-pilots, whereas military flights are usually piloted by only one.
e. Recently, the number of military pilots also piloting commercial airliners has declined.
The correct answer is (C). The argument concludes that the reason for the military's decision was to reduce pilot error during
commercial flights. (C) is the only answer choice that supports this conclusion. Given that chemical warfare is likely to escalate in
the future, it would seem that the military would continue to require immunization shots. But the military stopped requiring the
shots. So the military's decision must have been based on some factor outweighing the potential danger of chemical warfare to
pilots. One such possible factor is the increased danger of commercial airline accidents resulting from the immunization shots.
I am very confused by this explanation. My answer of last resort was B, which I thought made the most sense out of any of them...
unfortunately, it does.
remember that "provides the most support" means just that: the most support. if all four of the other answer choices provide zero
support, then a fifth answer choice that bolsters the argument at all, no matter by how little, wins by default.
--
according to the passage, a significant # of pilots fly both military and commercial aircraft. therefore, the idea of the problem is that
we have to figure out which concerns - commercial vs. military - lie at the heart of the quoted policy.
therefore:
if a quoted reason points to the idea that the policy is motivated by commercial concerns, or NOT motivated by military
concerns, then it strengthens the argument.
if a quoted reason points to the idea that the policy is motivated by military concerns, orNOT motivated by commercial
concerns, then it weakens the argument.
--
if (d) is true, it supports the idea that the policy is aimed at retaining military pilot. this would mean that military, not commercial,
concerns lie at the heart of the policy, weakening the argument.
if (b) is true, then pilots' dizzy spells will have less of an effect on commercial flights, because of the presence of co-pilots. this
would mean that commercial concerns are less important to the policy than are military concerns, weakening the argument.
this leaves (c) by process of elimination, since all of the other choices are either irrelevant or detrimental to the argument.
(c) is the best choice anyway, because it shows that military concerns DO NOT motivate the policy, or, at least, that the policy is at
odds with clear military objectives (cancelling anti-chemical-warfare immunizations vs. escalating chemical warfare).
as noted above, this strengthens the argument, because it must then be true that commercial concerns DO motivate the policy.
try to revrse the options and see the effects ...even if u get
something to conlude from premise....conclusion must be playing
a role...jsut that u need to connect
from plate boundaries and contain only minor faults. Since no minor fault in a geologically quiet region produces an
earthquake more often than once in any given 100,000-year period, it follows that of all potential nuclear reactor sites in
such a region, those that are least likely to be struck by an earthquake are ones located near a fault that has produced an
(A) Geologically quiet regions are the least dangerous regions in which to build nuclear reactors.
(B) For any potential nuclear reactor site, the likelihood of being struck by an earthquake is the primary determinant of site
safety.
(C) In a geologically quiet region, every potential nuclear reactor site is near at least one minor fault.
(D) Nuclear reactors that are located in geologically quiet regions are built to withstand at least one but not necessarily more
(E) Earthquake faults in geologically quiet regions produce earthquakes at least once in 100,000 years.
I understand when people crib about RC and SC. But the truth is that CR is not hard to rule. All you need is some patience and
correct guidance.
You need to find the assumption here. An assumption is a missing premise that is necessary for the conclusion to hold - again, it
is necessary for the conclusion. Look for the conclusion of the argument.
Conclusion: Of all potential nuclear reactor sites in such a region, those that are least likely to be struck by an earthquake are
ones located near a fault that has produced an earthquake within living memory
(A) Geologically quiet regions are the least dangerous regions in which to build nuclear reactors.
The conclusion categorically states: "Of all potential sites in such a region" i.e. it is only considering quiet regions. It is not
comparing geologically quiet regions with other regions so this option is out of scope.
(B) For any potential nuclear reactor site, the likelihood of being struck by an earthquake is the primary determinant of site
safety.
Again, the conclusion does not talk about 'site safety'. It only discusses the likelihood of an earthquake. So again out of scope.
(C) In a geologically quiet region, every potential nuclear reactor site is near at least one minor fault.
The conclusion says that of all potential sites in a quiet region, the safest (regarding earthquakes) is the one near a minor fault
that has recently produced an earthquake. It does make sense in a way since a recent earthquake would have settled the plates
and hence it is not very likely that another earthquake will happen anytime soon. But the question is - 'Is it necessary that there
will be a minor fault near all the potential sites?' How about a site which has no faults and hence no chance of an earthquake?
Hence, when we say, 'Build the plant near a minor fault which has seen an earthquake in the recent past,' we are assuming that
there is no site which has no faults. We are assuming that every potential site has at least one minor fault. Answer (C)
(D) Nuclear reactors that are located in geologically quiet regions are built to withstand at least one but not necessarily more
than one earthquake of minor to moderate force.
Again, it is out of scope since the conclusion does not talk about how many earthquakes the reactor can stand. It is only trying to
find the site where chances of an earthquake are minimum.
(E) Earthquake faults in geologically quiet regions produce earthquakes at least once in 100,000 years.
We don't need to assume this to make the conclusion true. 1 in a 100,000 is just the average. There needn't be 1 in every 100,000
yrs. The conclusion is only trying to find the most stable area to make the reactor. A fault that has experienced an earthquake
recently is expected to be more stable that a site with a minor fault which hasn't experienced an earthquake lately.
A veterinary pharmaceutical manufacturer implemented a business strategy to encourage sales by creating a product that could
be legally sold over the counter, without the need for a prescription from a veterinarian. The product was a collar for cats
intended as a repellent against fleas that, through biting the host animal's epidermis, would die after ingesting some of its blood.
Despite the advantage of not having to consult a veterinarian in order to buy one, the collar was not successful commercially.
Which of the following, if true, does the most to explain why the manufacturer's strategy failed to achieve its objective?
A) Although the collar was only to be worn externally, its chemical components, through constant contact with the animal's
skin, would be absorbed into the bloodstream, not only killing newly-attached fleas, but also ceasing the reproductive cycle
of already present fleas and eggs.
B) It has been proven that pet products with recommendations made by veterinarians printed on their packaging sell far
more successfully than those that do not have such recommendations printed on their packaging.
C) The sales of flea collars and other repellents used to maintain pet health are greatly affected by the changes of season,
usually very low during the winter and autumn months of the year, rising with the beginning of spring.
D) To be able to sell non-prescription products with active ingredients such as propuxer, the chemical used in anti-pest
products, legislation requires that manufacturers limit the presence of the active ingredient.
E) Consultation with a professional veterinarian is always advisable when confronting an issue regarding an animal's health
since even someone with a fair amount of medical knowledge may not be aware of illnesses or ailments associated with a
specific animal.
A veterinary pharmaceutical manufacturer implemented a business strategy to encourage sales by creating a product that could
be legally sold over the counter, without the need for a prescription from a veterinarian. The product was a collar for cats
intended as a repellent against fleas that, through biting the host animal's epidermis, would die after ingesting some of its blood.
Despite the advantage of not having to consult a veterinarian in order to buy one, the collar was not successful commercially.
Which of the following, if true, does the most to explain why the manufacturer's strategy failed to
the animal's skin, would be absorbed into the bloodstream, not only killing newly-attached fleas, but also
B) It has been proven that pet products with recommendations made by veterinarians printed on their
packaging sell far more successfully than those that do not have such recommendations printed on their
packaging.
C) The sales of flea collars and other repellents used to maintain pet health are greatly affected by the
changes of season, usually very low during the winter and autumn months of the year, rising with the
beginning of spring.
D) To be able to sell non-prescription products with active ingredients such as propuxer, the chemical used in
anti-pest products, legislation requires that manufacturers limit the presence of the active ingredient.
E) Consultation with a professional veterinarian is always advisable when confronting an issue regarding an
animal's health since even someone with a fair amount of medical knowledge may not be aware of illnesses or
Source: gmat.babson.edu
Responding to a PM
Option B is incorrect because printing recommendations on packaging is different from requirement of prescriptions. Option B is
actually out of scope. Just because a vet product does not require prescription before use, it does not mean that it cannot be
recommended by vets. For example: there are many medicines available over the counter, which are recommended by the
doctors. So, whether pet products recommended by vets are more successful or not does not affect the argument.
Option D is correct for the reason stated by Vishnu above. However, this option is also not GMAT like. The clarity that I find in
correct GMAT options is not at all there in this option statement. Are fleas considered pests? Does limiting the active ingredient
affect the performance of the product? These are important questions but unanswered.
Thanks,
Chiranjeev
ALWAYS BE WARY OPTION HAVING WORD PLAY ...THEY WILL CATCH U RIGHT THERE
16. A reason Larson cannot do the assignment is that she has an unavoidable scheduling conflict. On the other
hand, a reason Franks cannot do the assignment is that he does not quite have the assertiveness the task
requires. So, the task must be assigned to Parker, the only supervisor in the shipping department other than
(B) The task cannot be assigned to anyone other than a supervisor in the shipping department.
(C) Franks would be assigned the task if Franks had the assertiveness the task requires.
(D) The task cannot be assigned to anyone who has any kind of scheduling conflict.
(E) No one who is not a supervisor in the shipping department has the assertiveness this task requires.
Argument:
Larson cannot do the assignment.
Franks cannot do the assignment.
Parker is the only other supervisor in the shipping department.
(B) is the correct assumption - The task cannot be assigned to anyone other than a supervisor in the shipping department. We are
certainly assuming that since Parker is the only other supervisor, he must do the assignment. Only a supervisor in shipping dept
can do the assignment.
(D) The task cannot be assigned to anyone who has any kind of scheduling conflict.
This is not correct. Anyway, it says 'any kind of scheduling conflict' while the premises mentioned that Larson has 'unavoidable
scheduling conflicts'. This implies there might be some scheduling conflicts which can be readjusted/avoided etc.
Even if the option had said "The task cannot be assigned to anyone who has an unavoidable scheduling conflict.", it would still
not be an assumption because it is already given in the argument. So it is not a missing premise and hence not an assumption.
Answer (B)
WHENEVER THERE IS PERCENTAGES OR SOME DATA CR QUETSION ASSUME 100 AND START
PUTTING NUMBER STO GETH THE RIGHT PERPECTIVE...AND NEVER HURRY IN CR INITIAL
READING AS THEN U HAVE TO READ TWICE OR THRICE......WRITE LESS AND UNDERSTAND
MORE
thelosthippie wrote:
Epidemiologist: In Brazil, schistosomiasis and malaria are the two most common infectious diseases. Over the past five years,
schistosomiasis has been primarily responsible for the overall increase in infectious disease cases. Therefore it is likely that there
A. Over the past five years, the percentage of infectious disease cases caused by schistosomiasis has remained unchanged, but
C. Five years ago, many more people died from schistosomiasis than died from malaria.
D. Over the past five years, the percentage of infectious disease cases caused by malaria was not significantly greater than the
E. Over the past five years, the percentage of infectious disease cases caused by diseases other than malaria and schistosomiasis
A. Over the past five years, the percentage of infectious disease cases caused by schistosomiasis has remained unchanged, but
the percentage of cases caused by malaria has increased substantially.
If most of the increase in cases is due to schistosomiasis but the % of schistosomiasis in the total cases is the same, it means,
most of the cases comprised of schistosomiasis earlier too. So this strengthens our conclusion. Let's take a look using numbers
too.
Today if still 70% infectious disease cases are those of schistosomiasis, it means there are 105 schistosomiasis cases today (an
increase of 35 out of the extra 50).
If malaria cases have increased to 30%, number of malaria cases would be 45 i.e. an increase of 15 out of the extra 50.
We see that schistosomiasis cases account for most of the extra 50 cases i.e. they account for 35 cases. So this strengthens our
conclusion.
B. Trends in infectious disease data tend to persist for decades, unless there is a major breakthrough in treatment regimens.
We don't know if there has been a major breakthrough in treatment regimens on any disease in the past 5 yrs. It is possible that 5
yrs ago there were more cases of malaria than schistosomiasis but breakthrough in treatment regimen of malaria led to a decline
in its cases.
C. Five years ago, many more people died from schistosomiasis than died from malaria.
Number of deaths is irrelevant to the number of cases since the argument doesn't say which disease is more serious, has better
medicines etc. It is possible that schistosomiasis had fewer cases 5 yrs ago but is much more serious and led to death more
often.
D. Over the past five years, the percentage of infectious disease cases caused by malaria was not significantly greater than the
percentage of cases caused by schistosomiasis.
This says that % of malaria cases was not much greater (though perhaps a little greater) than % of schistosomiasis cases over the
past 5 years. If anything, this weakens our conclusion. It indirectly implies that out of 100 cases 5 yrs ago, malaria cases were
slightly more than schistosomiasis cases.
E. Over the past five years, the percentage of infectious disease cases caused by diseases other than malaria and schistosomiasis
did not increase substantially.
This says that % of other diseases has held steady. So if earlier, 10% of cases (10 cases) were other diseases, still 10% of cases
(i.e. 15 cases) are other diseases. This doesn't say whether 5 yrs ago there were more cases of schistosomiasis or malaria.
Answer (A)
_________________
Karishma
Veritas Prep | GMAT Instructor
My Blog
SOFT WORDING FOR INFERENCE ALWAYS
APPRECIATED ...TRY TO SEE ADIFFERENCE IN
WORDING N U WILL GET IT FIRST GET THE RELATED
ONESSS.....AND THEN START ELIMINATING OMNG
THEM
TWO THINGS ALWAYS DO THE WORD
VERIFICATION FOR ANY SEEMINGLY FALSE OPTION
U FEELL JSUT CHECK VERY CAREFULLY THATS THE
TRICK ....MILD WORDING MUST ALWAYS APPEAL U
AND SHOULD BE CHECKED TWICE
Traditionally, candidates for elected offices have concentrated their efforts on the early stages of the campaign during which,
most people believed, the voters' perceptions of candidates were formed. It is now becoming clear, however, that elections can
be decided in the few days preceding election day; public opinion polls taken during recent elections provide evidence of
several such races. In those cases, the losing candidates would have been well advised to have forgone early spending and
instead saved funds for television advertisements late in their campaigns.
Which of the following inferences can be most reasonably drawn from the information in the passage above?
A) Television has had an adverse effect on political campaigns, making them less issue-oriented.
B) Politicians of the pre-television era fail to understand the important role television advertising plays in today's political
campaigns.
C) Public opinion polls often inaccurately reflect the mood of the electorate in the early stages of a political campaign.
D) Polls taken in the days preceding a major election may not accurately predict the outcome of an election.
E) Candidates should not try to define the key issues of a race until late in the campaign.
The argument is similar to the following link, but the question stem and options are different here:
TirthankarP wrote:
Traditionally, candidates for elected offices have concentrated their efforts on the early stages of the campaign during which,
most people believed, the voters' perceptions of candidates were formed. It is now becoming clear, however, that elections can
be decided in the few days preceding election day; public opinion polls taken during recent elections provide
evidence of several such races. In those cases, the losing candidates would have been well advised to have forgone early
spending and instead saved funds for television advertisements late in their campaigns.
Which of the following inferences can be most reasonably drawn from the information in the passage above?
A) Television has had an adverse effect on political campaigns, making them less issue-oriented.
B) Politicians of the pre-television era fail to understand the important role television advertising plays in today's political
campaigns.
C) Public opinion polls often inaccurately reflect the mood of the electorate in the early stages of a political campaign.
D) Polls taken in the days preceding a major election may not accurately predict the outcome
of an election.
E) Candidates should not try to define the key issues of a race until late in the campaign.
note...... opinion polls are the ones taken during election..... these are taken as presumably correct.......
therefore the opinion polls taken before elections MAY be may not be correct.................
hence "d".......
ALWAYS KEEP A SLIGHTLY DOUBTFULL ANSWER FOR A BCKUP TO FINALLY COMPARE AND
GIVE ANSWERS.....DO NOT GO WITH UNRELATED CONCEPTS WHICH U THINK CAN GO WITH
SOME ASSUMPTIONS
Major airlines will purchase many of the new aircrafts capable of carrying more than 500 passengers on transcontinental and
transoceanic flights. These airlines currently rely on "hub and spoke" systems of routing, in which large planes, which can seat
400 people and are capable of transoceanic flight, fly into hubs that have runways sufficiently long to handle them. From there,
passengers are dispatched to local airports on connecting flights on small planes. With takeoff and landing time slots almost
completely booked at most hubs, and little new runway construction expected, airlines will want to expand the volume of
passengers they can fly in a given time slot.
AGAIN HERE U CAN WEAKEN THE IDEA OF 500 PASS PLANE....OR INTRODUCE A NEW IDEA
WHICH CAN BETTER ACHIEVE IT BEWARE OF ....WORD VERIFICATION IN OPTION WITH
PASSAGE..............A NEW TERMINOLY THOUGH A COMMON SENSE STILL SHOULD NEVER
NBE ACCEPTED......WHO KNOW MAY BE BORADING TIMES EVEN WHEN EXPANDED ONLY
TAKE 10 PERCENT OF THE TIME BETWEEN AND LANDING AND TAKE OFFF........ALWAY TRY
TO REASON OPPOSITEKY AND NOW FOR IT MOST IMPORATNT
The argument above would be most weakened if which of the following were true?
A. The new 500 seat aircraft cost more per seat than existing aircraft
B. Air traffic control systems at most hub airports cannot handle any more flights per hour than they currently do
C. The new 500 seat aircraft require boarding times substantially longer than those of existing aircraft
D. Small passenger aircraft, capable of efficient transcontinental and transoceanic flight and able to land on short runways, have
come into service
E. Transoceanic air flights are currently running at near maximum capacity
Please provide answer along with your reasoning for each option. Thanks.
A. The new 500 seat aircraft cost more per seat than existing aircraft
Irrelevant and out of scope. Hence out.
B. Air traffic control systems at most hub airports cannot handle any more flights per hour than they currently do
This one would look very attractive. However, this is something that is already mentioned in the argument. Read the last
sentence, With takeoff and landing time slots almost completely booked at most hubs, and little new
runway construction expected. Its already given that the most slots are booked. Also, as a remedy, there is a new
runway being setup. So this cannot weaken the argument. Hence out.
C. The new 500 seat aircraft require boarding times substantially longer than those of existing aircraft
Irrelevant and out of scope. Hence out.
D. Small passenger aircraft, capable of efficient transcontinental and transoceanic flight and able to land on short runways,
have come into service
This option weakens the argument. Our entire argument is based on the assumptions that there are no small aircrafts available
that are efficient in making transcontinental and transoceanic flight. Because, if there are such aircrafts then the Airlines will
buy these small aircrafts (instead of the bigger aircrafts) capable of making the flight and landing at the local airports directly.
This would increase the airllines volume of passengers at a given time slot. Hold
E. Transoceanic air flights are currently running at near maximum capacity
This does not weaken the conclusion in any way. Hence out
Hence D is correct.
A new species of fish has just been discovered living in great numbers in the waters off Papua New Guinea. The fish exhibits
some characteristics belonging to sharks, such as a cartilaginous skeleton. However, the fish also exhibits characteristics
belonging to eels, such as a long, snake-like body. Because of the habitat where it was found and its obviously eel-like body,
almost all scientists believe that the fish is an eel. But the cartilaginous skeleton puzzles them, since no known eel possesses
one. So scientists are still unsure as to the fish's precise classification, but they agreed immediately that the most logical
classification would be as either a shark or an eel.
The statements above, if true, most strongly support which of the following?
A. To be placed into a certain classification, a fish must possess all the characteristics of that classification.
B. Physical characteristics, such as skeletons, are scientists' primary means of classifying new species.
C. Some fish with cartilaginous skeletons are not sharks.
D. The waters off Papua New Guinea are generally hospitable to eels.
E. A fish cannot be both a shark and an eel
Conclusion: This is an inference question that asks us to draw a conclusion from the information provided. What must also be
true given the stimulus?
Evidence: Since we need to draw our own conclusion, we can use all facts provided in the argument as evidence.
The first sentence states that the new fish was discovered off the waters of Papua New Guinea. In addition, the sentence 4
states that the new fish shares its habitat with the eel. In combination, these two sentences indicate that eels also live in the
waters off Papua New Guinea. So this region must be hospitable to eels, as stated in (D).
(A) doesn't follow from the information given. The last sentence implies that the new fish could be classified as a shark even
though it does not have a cartilaginous skeleton Also, it is extreme in its use of the word all. With (B), the importance of physical
characteristics relative to other characteristics is not discussed. With (C), if the new fish were to be classified as an eel, then we
could conclude that at least one fish with a cartilaginous skeleton is not a shark. The stimulus, however, never says this.
Furthermore, we aren't told anything about other fish classifications. And with (E), the last sentence states that the new fish will
probably be classified as either a shark or an eel, but suggests nothing about an animal being both. Though this choice may seem
true, we don't know for certain.
READ THE KEYWORD AND SCOPE IDENTIFY CONCLUSION AND WHAT ANSWER IS ASKIG
...NIW SEE THE KEYWORD AND SCOPE OF FINAL 2 CHOICE...AND MOST IMPORATNATLY
QUESTION THEM ...RATHER THAN CHOOOSE N U WILL GET IT !!
Addressed to an Instructor
Advocates argue that five-cent bottle deposits charged on beverage containers are necessary for environmental protection because
they help to ensure that plastic and glass bottlesas well as aluminium cans are recycled .This is , the advicates say,because the five-
cent redemption programs provide a strong incentive to return the used containers to recycling facilities .However, a recent study
found that states without a bottle deposit had more success in implementing comprehensive recycling programs , which include
paper,plastics , and steel , in addition to the beverage containers , than did states with a bottle deposit law .
The answer to which of the following questions would be most useful in analyzing the significance of the study referenced above ?
(A) Did any of the states surveyed losr revenue on the bottle deposit program ?
(B) Do the citizens of the states that were studied prefer 5 cent redemption programs on beverage containers ?
(C) When the 5 c cent deposit programs were implemented , were the citizens of the states the began deposit programs as
enthusiastic about recycling as the citizens of the other states ?
(D) Did the citizens of the states with and without bottle deposit programs purchase compare number of beverages in plastic,glass
and aluminium containers ?
(E) Were the bottle deposit and the comprehensive recycling programs given equal funding ?
OA C
I MEntioned the OA
I got a PM on this question, which is fortunate, because I wrote this one a while back...
Firstly, it is not really a plan question. Yes there is a plan involved, but the question is what do I need to know to evaluate the
significance of a study? So this is more of a "most useful to know" type of question. As in, "which of the following is the most useful
to know when evaluating the relevance of this study to this conclusion?"
Now the study found that the states without bottle deposit laws did better in overall "comprehensive" recycling programs than did
those states with bottle deposit laws. The implied conclusion is that it is a better plan to NOT have a bottle deposit law if your goal
is comprehensive recycling.
Now the answer choices a variations on the theme of -- what could be the alternate cause for the success of the states without the
laws? Might it just be coincidence that they are performing better? In other words the states without bottle deposit laws might have
something else in common that allows them to be more successful in comprehensive recycling.
Now C would be useful to know. Basically, "are the citizens in states with and without bottle deposits equally enthusiastic?" If they
are not then that could be a big cause of the difference noted in the study. Maybe that is the reason some states have more
success than others.
E also seems to be a good choice. Funding is a big deal in the success of a program. I meant for E to be worded in such a way that
it would be taken out of the equation. It says, "Were the bottle deposit and the comprehensive recycling programs given equal
funding?" Now this was supposed to not be an equal comparison. Bottle deposit focuses only on beverage containers while the
comprehensive focuses on everything recyclable. So this is meant to not be as useful because it is apples to oranges, not a true
comparison.
D is meant to be a distraction as well. It should not be important that they citizens purchase equal number of beverages, instead
what they do with those containers should be key.
So approach as a most useful to know and see what information helps you to decide whether you can make conclusions from this
study.
_________________
Argument presents a paradox and the paradox is: "Why a minimum reserve price is set for the most desirable bids". Argument
assumes that if it is "desirable" most bidders will try n present the highest quote so as to get the their desirable bid.
E is the correct answer because it brings into light the behavior of prospective bidders for such desirable bids. this behavior is
that there may be many other bidder who may or may not be prospective bidder but since the bid is most desirable they will bid
a very high quote and hence it may be not economical. So these prospective bidders may not even bid.Hence a limit or the least
quote price is needed for most desirable bidders.
Argument presents a paradox and the paradox is: "Why a minimum reserve price is set for the most desirable bids". Argument
assumes that if it is "desirable" most bidders will try n present the highest quote so as to get the their desirable bid.
E is the correct answer because it brings into light the behavior of prospective bidders for such desirable bids. this behavior is
that there may be many other bidder who may or may not be prospective bidder but since the bid is most desirable they will bid
a very high quote and hence it may be not economical. So these prospective bidders may not even bid.Hence a limit or the least
quote price is needed for most desirable bidders.
noboru wrote:
16. Publicity campaigns for endangered species are unlikely to have much impact on the most important environmental
problems, for while the ease of attributing feelings to large mammals facilitates evoking sympathy for them, it is more difficult
to elicit sympathy for other kinds of organisms, such as the soil microorganisms on which large ecosystems and agriculture
depend.
Which one of the following is an assumption on which the argument depends?
Conclusion: Plublicity compaign will not much impact the most important environmental problems.
Now I will use denial method to test if negation of the answer choice undermines the conclusion. If it undermines, the answer
choice will be the correct assumtion.
(A) The most important environmental problems involve endangered species other than large mammals.
==> The negation of the choice is "The most important environmental problem includes only the large mammals."
Now since large mammals evoke sympathy easily, publicity compaign must have impact on the most important
environmental problems. This clearly undermines the conclusion and hence it is correct assumption.
(B) Microorganisms cannot experience pain or have other feelings.
==>Irrelevent
(C) Publicity campaigns for the environment are the most effective when they elicit sympathy for some organism.
==> "the most effective" is too extreme. The conclusion talks about some impact not the most effective way to
impact.
(D) People ignore environmental problems unless they believe the problems will affect creatures with which they sympathize.
==>The negation of the choice is "People do care about environmental problems even if it affect the creature they do
not sympathize". This has no effect on the conclusion as we are not sure what the most important environmental
problem involves.
(E) An organism can be environmentally significant only if it affects large ecosystems or ag
students will need in higher grades to develop the ability to solve complex problems. Learning basic math skills is like learning
the scales and chords that one will later use to master complicated concertos and symphonies. However, math educators in this
country seem to have it backward, emphasizing in higher grades the same narrow, skills- based approach that students learned in
lower grades rather than the analytical tools they will need to solve complex math problems.
Which of the following, if true, would most seriously weaken the conclusion drawn above?
AGAIN QUESTION AGAINST EACH LOOOSE TERM ATLEAST ....AND GAPS IN CHOICESS...DO NOT TAKE IT
POSITIVELY ..C AND E ....DIFFERENR ANSWER FROM LOWER CAN MEAN ANYTHING....SO NOT A CONVINCING
ANSWER
(A) While music is common in elementary school curricula, it is rarely taught in high school.
(B) On international tests of math skills, high-school students in this country performed no worse than did their counterparts
(C) When presented with a math problem to solve, students in higher grades are more likely to arrive at different answers than
(D) Older students tend to receive higher grades in math than do younger students.
(E) Universities in this country report a steady increase in the percentage of native first-year students who qualify to take
Dear nikhil.jones.s,
I'm happy to help. I'm not sure that I like this question. What is the source? Among other things, the plural of "curriculum" is
"curricula", not "curriculums."
Here, the conclusion is the first sentence: "Math education in this country does a disservice to our children." The rest of the
argument provide evidence. We would most seriously weaken the conclusion by demonstrating that the educational system
serves math students well.
(A) While music is common in elementary school curricula, it is rarely taught in high school.
Pure distractor. Music is mentioned as an analogy, but it's irrelevant to the thrust of the argument. This is incorrect.
(B) On international tests of math skills, high-school students in this country performed no worse than did their counterparts
from countries where problem-solving is emphasized in higher grades.
Tempting, but if the other countries are also poorly serving their math students, and everyone is at a low level together, then it
could still be that the country discussed in the prompt argument does not serve its students well. This is incorrect.
(C) When presented with a math problem to solve, students in higher grades are more likely to arrive at different answers than
students in lowers grades are.
Of course they do. If students of all grades solve the problem the exact same way, that would really show that students were
learning nothing. The fact that older students have a different approach and find different solutions could be a bad sign or it
could be hopeful --- depending on whether the older students were correct more frequently than the younger students. No clear
implication can be drawn. This is incorrect.
(D) Older students tend to receive higher grades in math than do younger students.
This compares apples to oranges. The grades that students receive at different levels are relative to each other, to creates in
other subjects at that level, etc. A direct comparison of the letter grades of two different grade levels does not make any sense.
This is incorrect.
(E) Universities in this country report a steady increase in the percentage of native first-year students who qualify to take
advanced mathematics courses such as calculus.
Aha! A calculus course in college provides a kind of standard or benchmark against which you can compare different cohorts of
students. If more first-year college students are qualifying for advanced calculus, that means they absolutely have to be learning
some good math! Therefore, the education system is serving them well. This decisively weakens the argument.
IF U FIND THAT U CAN COUNTER BOTH THE CHOICES ...THE SRIOULY CONSIDER THE OTHER
ONESSS...N U WILL CERTAIN;LY GET THE ANSWER FROM THE REST
IT SAYS MOST ACCURATELY EXPRESSES THE CONCLUSION DRAWN ...SO THE CONCLUSION IS
ALREADY DRAWN ...WHAT IS THE CONCLUION.............
THERE WILL BE THINGS IN THE PASSAGE WHICH WILL BE TRUE ...BUT THOSE THINGS
SDOESN'T MEAN THEY ARE ANSWERS RATHER SUCH REPETION OF THE PREMISES OR
CONCLUION WILL NEVER BE EXACT ENASWER...ANSWER HAVE TO BE SOMETHIN NEW ....AS
A GENERAL RULE U MAKE THIS MISTAKE AND GET TRAPED NEVER DO THIS MAN!!
Public health expert: Until recently people believed that applications of biochemical research would eventually achieve
complete victory over the microorganisms that cause human disease. However, current medical research shows that those
microorganisms reproduce so rapidly that medicines developed for killing one variety will
only spur the evolution of other varieties that are immune to those medicines. The most rational public health strategy,
therefore, would place much more emphasis than at present on fully informing people about the transmission of diseases
caused by microorganisms, with a view to minimizing the incidence of such diseases.
Of the following, which one most accurately expresses the conclusion drawn by the public health expert?
(A) A medicine that kills one variety of disease causing microorganism can cause the evolution of a drug-resistant variety.
(B) A patient who contracts a disease caused by microorganisms cannot be effectively cured by present methods.
(C) There is good reason to make a particular change to public health policy.
(D) No one who is fully informed about the diseases caused by microorganisms will ever fall victim to those diseases.
(E) Some previous approaches to public health policy ignored the fact that disease-causing microorganisms reproduce at a
rapid rate.
public health strategy would place much more emphasis <than at present> on fully informing people about blah blah
'than at present' tells us that there is scope of editing/revising the publick health strategy.
OR IF THERE ARE 2 REASON ...U WANT STRENTHEN THAT ONE IS THE REASON U CAN SAY
THAT 2 DOES NOT EFFECT IS NOT THE REASON....SO U CAN SAY IT WAS THERE ALL THE
TIME BUT NO EFFECT!!
1. Volcanoes
Now B Says ,Massive volcanic eruptions occurred frequently 80 million years ago.Moreover,nowhere it is mentioned
A. Volcanoes massive enough to generate high concentrations of iridium are very rare.
C. Most scientists support the hypothesis that a cosmic impact wiped out the dinosaurs.
D. The massive extinction that occurred 70 million years ago killed not only the dinosaurs but also 70 percent of all life on Earth.
E. A comet struck the earth some 120 million years ago, but no widespread extinction occurred.
Plz Advice !!
Look at the argument again - two things cause concentration of iridium - volcanoes and meteors
Volcanoes occured frequently while dinosaurs lived on Earth. Chances are that an activity which was not usual wiped out
dinosaurs. Since dinosaurs were wiped out 65 million yrs ago and there was iridium concentration in rocks which are from 65
million yrs ago, it makes it more likely that a meteor wiped out dinosaurs.
LAWAYS REMEMBER FIRST ELIMINATE ALLL THE CHOICES ATATING SAME STUFF AS GIVEN IN
THE PSSAGE ....FOR THE ASSUMPTION QUESTIONS/ STRENTHEN QUESTION ...OTHER
QUESTION WILLN NOT HAVE SUCH A TRAP IF IT IS A DECENT QUESTION ...........AND THEN
REMOVE THE OUT OF SCOPE ....AND THE USE NEGATION TO NAIL IT DOWN
Transcendentalist wrote:
In the late nineteenth century, the archaeologist Charles Warren discovered a new way to explore otherwise inaccessible areas,
using vertical shafts leading to horizontal tunnels deep beneath the surface. These tunnels led directly to those areas. Based on
Warren's discovery, Zachi Zweig, a rising star in the archaeological scene, has concluded that similar shafts may be dug on shores
in proximity to sunken archeological artifacts, leading to tunnels beneath the sea floor allowing easy access to those artifacts.
A) The specific weight of water and earth are similar, thus creating the same pressure on the tunnel's ceiling.
B) Otherwise inaccessible sunken archeological artifacts can be reached using new technological advancements in the world of
marine robotics.
C) Tunnels beneath the seafloor and tunnels beneath dry land share the same vertical distance from the surface.
D) Shafts similar to those dug by the celebrated Warren may be dug on the sea-shore, enabling archaeologists easy access to
E) Otherwise inaccessible terrain can be reached below the surface using vertical shafts. ............ALREADY GIVEN ELIMINATE
OE to follow
This question is a bit awkward as it's not a great representation of a GMAT question (it looks like it's from the Economist?)
Anyway, as the prior posts indicate, removing 3 of the 5 options is relatively easy (very common on CR questions) and we are left
with 2 options that both seem to be possible answers, A&C. On Critical Reasoning, you want to get rid of the garbage (obviously
wrong answers) quickly so you can spend time working on the 2 possible answers.
Let's dig into A&C. The premise of the argument presents an approach of digging deep vertical shafts connected to tunnels that
will reach previously unaccessible areas. The conclusion is that the same method can be used to reach sunken artifacts (under
water). What is the necessary assumption between A & C? Let's try negating...
Negated A - The specific weight of water and earth are not similar, thus creating different pressure on the tunnel's ceiling.
If there is different pressure between the tunnels under ground and under sea and there is more pressure under sea, might the
under sea tunnels collapse and prevent reaching the sunken artifacts? Very possibly.
Negated C - Tunnels beneath the seafloor and tunnels beneath dry land do not share the same vertical distance from the surface.
Does the distance to the surface impact the ability to dig shafts connected to tunnels? The premise states that the underground
tunnels are "deep" under the earth so it appears that depth doesn't impact the ability to use this technique.
Normally with negation we get the destruction of the conclusion. I wouldn't say that negated 'A' destroys the conclusion, but it
seems to do much more harm to the conclusion than negated 'C', so we will choice answer choice A.
KW
Hi swarman
The purpose of the argument is to find an explanation i.e result of a comparaison (Jacob must have watched more television as a
child than I did) to a present fact (Jacob is an obese adult, I am not an obese adult) based on a result of a study ( Studies show
that children who watch too much television are more likely than others to become obese adults)
A. The hardware store on Main Street must have had a bigger advertisement in the Sunday paper than the hardware store around
the corner had. The hardware store on Main Street sold twice as many items as the hardware store around the corner did last
week, and a Sunday paper advertisement has been shown to increase the number of items sold.
The purpose of this argument is to find an explanation i.e result of comparaison ( the HSMS must have had a bigger ads in the S.
PAPER than the HSAC) to a present fact (the HSMS sold twice as many items as the HSAC did last week) based on a result of a
study ( S. PAPER ADS have been shown to increase the num of items sold)
B. Studies show that large dogs lives shorter lives, on average, than small dogs do. Rex is a large dog and therefore might be
expected to live a shorter life than Mustang, who is a small dog.
The purpose of this argument is to predict an event on the future based on a result of a study
C.The county superintendent stated that all schools would be canceled for the day if snowfall last night were greater than six
inches. Therefore, since the snowfall was only five inches, we must be following the usual school schedule today.
The purpose of this argument is to explain the causal-effect of a general statement
D. According to research, people with unusual musical talent do not achieve their true potential unless they are given formal
lessons. Therefore Jesse, who has achieved his full musical potential without formal lessons must not have unusual musical
talent.
the purpose of this argument is to find an explanation to an event occured in the past based on a result of a research
E. People who like vegetables also like fruits. Elizabeth does not like fruits, so she must not like vegetables either.
The purpose of this argument is to predict something based on a general observation
Hence, A is the most closely parallels the logical structure of the original argument.
"If you don't change your life, your life will change you"
UNDERSTAND THE TWIST HERE ...IT SAYS MOST CANNOT BE DONE BECUZ ONLY ...A FEW
ARE USEFULL !!
The only preexisting recordings that are transferred onto compact disc are those that record companies
believe will sell well enough on compact disc to be profitable. So, most classic jazz recordings will not be
transferred onto compact disc, because few classic jazz recordings are played on the radio.
Conclusion:
Most CJRs will not be transferred onto CDs. WHY? because few CJRs are on the Radio(I think why there is a sudden addition of
radios into the picture)
So, taking the logical leap here, the argument is saying that
1)those Recs. that are played on the radio will be transferred onto CDs.
2)those recs. that are transferred onto CDs will bring $$$.
Hence, Recs. that are played on the radio will bring $$$ to company. precisely what E is saying.
(A) Few of the preexisting recordings that record companies believe can be profitably transferred to compact disc are classic jazz
recordings.
(B) Few compact discs featuring classic jazz recordings are played on the radio. stated in the argument. Hence, cannot be
conclusion.
(C) The only recordings that are played on the radio are ones that record companies believe can be profitably sold as compact
discs. false. This statement is the same as St.1 of the stimulus, with Pre exist. recs. replaces by Radio. the conclusion does not
link PERs and Radio.
(D) Most record companies are less interested in preserving classic jazz recordings than in making a profit. Irrelevant.
(E) No recording that is not played on the radio is one that record companies believe would be profitable if transferred to
compact disc. [color=#FF0000]This statement implies that a recording that is played on the radio is, in fact, the one that
companies believe is profitable. [/color]
In the country of Bedenia, officials have recently implemented a new healthcare initiative to reduce dangerous wait times at
emergency rooms in the countrys hospitals. This initiative increases the number of available emergency nurses and doctors in
urban settings: scholarships and no-interest loans are being offered to prospective students in these fields if they work in major
city hospitals, relocation packages to urban centers are being offered for current emergency practitioners, and immigration rules
are being changed to enable foreign emergency doctors and nurses to more easily move to Bedenias major cities.
Which of the following would be most important to determine in assessing whether the initiative will be successful?
B. Which hospitals in Bedenia have dangerous wait times in their emergency rooms
C. Whether a career in emergency medicine pays substantially less than other types of medicine
D. Whether wait times could be reduced by means other than increasing the number of available nurses and doctors
E. Whether many foreign doctors and nurses are currently not allowed to enter Bedenia
Good question thelosthippie +1 kudo.
KEY is: "....reduce dangerous wait times at emergency rooms in the countrys hospitals". ==> Not only urban area. What if
dangerous wait times happen mainly in rural areas. But the initiative just try to increase doctors and nurses in major city
hospitals. So we need to determine which hospitals in Bedenia have dangerous wait times in their emergency rooms
B is correc
IN ANY QUESTION WITH STATS AND DATA ....JUST UNDERSTAND THE DATA ...AND MOST
IMPORTANLY PUT DOWN THE CONSITIONS OF THE DATA USING GRETER THAN AND LESS THAN
SIGNN...IN A NICE CLEAR TABULATED MANNER ..IF U WANT TO ASSUME A SUMM AS 100 SO THAT
BUT CONVERT WORD CONDITIONS TO NUMBER VERY C;EAR;Y ....AND THEN MOVE TO OPTIONS ...U
WILL GET IT ALWAYS
In Patton City, days are categorized as having heavy rainfall (more than two inches), moderate rainfall (more than one inch, but
no more than two inches), light rainfall (at least a trace, but no more than one inch), or no rainfall. In 1990, there were fewer
days with light rainfall than in 1910 and fewer with moderate rainfall, yet total rainfall for the year was 20 percent higher in
1990 than in 1910.
If the statements above are true, then it is also possible that in Patton City
(A) the number of days with heavy rainfall was lower in 1990 than in 1910
(B) the number of days with some rainfall, but no more than two inches, was the same in 1990 as in 1910
(C) the number of days with some rainfall, but no more than two inches, was higher in 1990 than in 1910
(D) the total number of inches of rain that fell on days with moderate rainfall in 1990 was more than twice what it had been in
1910
(E) the average amount of rainfall per month was lower in 1990 than in 1910
quantum wrote:
In Patton City, days are categorized as having heavy rainfall (more than two inches),
moderate rainfall (more than one inch, but no more than two inches), light rainfall (at
least a trace, but no more than one inch), or no rainfall. In 1990, there were fewer days
with light rainfall than in 1910 and fewer with moderate rainfall, yet total rainfall for the
If the statements above are true, then it is also possible that in Patton City
A. the number of days with heavy rainfall was lower in 1990 than in 1910
definitely possible. Consider in 1990 one day about 1000000000 inches fell. (just kidding, but u get the point)
B. the number of days with some rainfall, but no more than two inches, was the same
in 1990 as in 1910 can't be true because we know moderate and light says were fewer in 1990 than in 1910
C. the number of days with some rainfall, but no more than two inches, was higher in
D. the total number of inches of rain that fell on days with moderate rainfall in 1990
was more than twice what it had been in 1910 not mathematically possible. We can only get twice the amount of rainfall
if the number of moderate days were the same and all the moderate days in 1990 have 2 inches of rain and all the
E. the average amount of rainfall per month was lower in 1990 than in 1910
not possible (unless there were more months in 1990 )since we are told total in 1990 was hight
to solve any environmental problem that is not the result of government mismanagement------> major changes in consumer
habits
NO economically enticing , no changes in environmental problem that lie in consumer habit
so, under this condition the only problems that will be solved are those related to government mismanagement
(A) Few serious ecological problems are the result of government mismanagement.
(B) No environmental problems that stem from government mismanagement have solutions that are economically feasible. we
have no information about the economic feasibility of problems that stem from government mismanagement
(C) Major changes in consumer habits can be made economically enticing. no new information. this info is already mentioned in
premise. assumption answer choice should contain new information+must be true+support the conclusion
(D) Most environmental problems that are not the result of government mismanagement are major ecological problems. shell
game!! in the argument we have major changes and environmental problems not major environmental problems
(E) Few serious ecological problems can be solved by major changes in consumer habits.
we have major changes in consumer habits will occur only if such changes are economically enticing, and w/o economic
enticingfew serious ecological problem can be solved unless the solutions are made economically enticing. this answer choice
combines two parts of two different premises into one statement.
FOR ANY STRENTHENER QUESTION IDENTIFY THE ASSUMPTION ...NOW A STRENTHENER
WILL BE SUPPORTING THE PREMISE OR ASSUMPTION OR ELIMINAYING A POSSIBILTY OF A
WEAKENER ......IN A QUESTION WITH NO CAUSE EEFECT
A PERSON PAST IS BAD DOES NOT MEAN HIS PRESENT HIS BAD SYCH EASONING SHOULD BE
AVOIDED IT IS TRIVIAL REASONING
Psychologist: Though popularized by the media and even some ill-informed scientists, photographic memory is a myth. Most of
the top competitors at this summers national memory championship have brains that are neurologically indistinguishable from
those of the general population and cognitive abilities that are seldom more than a standard deviation above average.
The competitors at this summers championship are typical of the segment of the population thought to have photographic
memory.
Strengthens - This option eliminates the possibility that the psychologist was using a wrong sample.
People with photographic memories would have brains that are neurologically distinguishable from those of the general
population.
Strengthens - This option validates the truth of Premise 2.
Photographic memory requires cognitive abilities more than a standard deviation above average.
Strengthens - This option validates the truth of Premise 3.
Photographic memory was initially proposed by a scientist whose theories of mind have since been widely discredited.
SLIGHT STRENTHEN OR NO STRENGTHEN THIS CANNOT STRENGTHEN JUST BECAUSE HIS OLD THEORIES ARE WRONG DOESNT
MEN THIS THEORY WILL BE WRONG ...SUCH PARRALEL SHOULD BE AVOIDE....
Photographic memory, if it exists, would be a requirement even to qualify for a national memory competition.
STRENTHENNER AS IT SAYS ....THERE IS NO SUCH QUALIFICATION REQUUIRED .....AND HENCE IT SUPPORT THAT IT IS A MYTH
....AS THE PARTICIPANTS IT SAYS ARE NORMAL AND SIMILAR TO OTHER PEOPLE
is it E
Gian wrote:
Some say that an asteroid colliding with Earth was responsible for the extinction of the dinosaurs. It is indeed likely that such a
collision did occur and that it caused the death of the vast majority of dinosaurs. However, new evidence suggests that perhaps
tens of thousands of dinosaurs lived far enough from the point of collision that they were not directly killed by the impact or by
the drastic change in temperature when immense dust clouds created by the impact blocked the sun. Some other force must
a) The water supply used by the dinosaurs who survived the asteroid collision and subsequent change in global temperature had
b) A second asteroid impact or some other cosmic event is likely to have killed off the dinosaurs that survived the asteroid
c) At least a few dinosaurs continued to exist for some years after the asteroid collided with Earth.
d) There is another existing theory that does a better job of explaining the extinction of the dinosaurs than the theory involving
e) The change in temperature caused when dust clouds from the asteroid impact blocked the sun could not have lasted for
several years.
CONCLUSION:
Someotherforce must have been responsible for the death of these remaining dinosaurs.
a) The water supply used by the dinosaurs who survived the asteroid collision and subsequent change in global temperature
had not become contaminated by lethal fallout from the collision.==>THIS SHOULD BE THE ASSUMPTION AS IF WE NEGATE
THIS:
The water supply used by the dinosaurs who survived the asteroid collision and subsequent change in global temperatureHAD
become contaminated by lethal fallout from the collision.
THEN IN THIS CASE IT SAYS IT WAS THE EFFECT OF ASTEROID COLLISION ONLY WHICH LEAD TO DEATH OF DINOSAURS.
b) A second asteroid impact or some other cosmic event is likely to have killed off the dinosaurs that survived the asteroid
impact and global temperature change.
IT IS TOO SPECIFIC TO SAY THAT SECOND ASTEROID IMPACT OR COSMIC EVEN LED TO DEVASTATION....this cant be assumption
c) At least a few dinosaurs continued to exist for some years after the asteroid collided with Earth.
not necessary it could also possible that during asteroid impact some other factor also came in to play leading to death of
dinosaurs.hence this cant be assumption.
d) There is another existing theory that does a better job of explaining the extinction of the dinosaurs than the theory involving
an asteroid collision with Earth.
this is out of scope we are not concerned whether there is some other theory or not we are just concerned that some other
factor led to death of dinosaurs
e) The change in temperature caused when dust clouds from the asteroid impact blocked the sun could not have lasted for
several years.
the duration of blocked sun doesnt affect the conclusion.
ALWASYS REMEMBER THE SCOPE AND THE PEOPLE INVOLVED INIT HERE
PSYCHOTHREAPIS ...ARE ATTEMPTING A TALK SHOW ...THAT NEED TO BE COUNTERED
....SO ITS TALKING ABOUT PSHYCHOTHEREPIST NOT DOING TALK SHOW .................BUT
PSYCHOTHERPY CAN BE PROVIDES BY ANY NOVICE ALSO .......WHICH MIGH HAVE A
CHNACE ....AS IN C ...SO THAT I S NOT MUST BE TRUE ............MUST BE TRUE IS A KEY
TRY TO SEE WORDS AND SCOPE AND IN THE LAST TWO OPTIONS TO NEIAL IT
My way in answering assumption questions is as follow :
i always ask my self " why (the conclusion) " ,,,,, so in this case the question will be :
WHY the psychotherapists should never provide psychotherapy on talk shows ??
the answer is the premise : because they will provide low quality info.
then create another question with opposite of the reason above : what if they
provide high quality ?
if the conclusion fall apart then you are close to the assumption.NOW FIND THIS IN
ANSWER CHOICES
if making high quality will harm the conclusion , then the assumption will be "they will never make high quality " which is a
restatement to answer E.
Psychologist: Psychotherapists who attempt to provide psychotherapy on radio or television talk shows are expected
to do so in ways that entertain a broad audience. However, satisfying this demand is nearly always incompatible with providing
high-quality psychological help.
For this reason, psychotherapists should never provide psychotherapy on talk shows.
Which one of the following principles must be assumed in order for the psychologists conclusion to be properly drawn?
(B) The context in which psychological help is presented has a greater impact on its quality than the nature of the advice that is given.
(C) Psychotherapy should never be provided in a context in which there is any chance that the therapy might be of less than high quality.
(D) Most members of radio and television talk show audiences are seeking entertainment rather than high-quality psychological help.
(E) Psychotherapists should never attempt to provide psychological help in a manner that makes it unlikely to be of high quality
WHILE READING TRY TO SPOT THE IDIOMS ALSO ...SO THAT U CAN ETLEAST
ELIMINATE THE THE OTHER OPTIONS USING IT
SEE THE REFERENCES AND MOST IMPORTANTLY THE MEANING BASED ON WHICHTHE
REFERENCES ARE MADE
Early 16th Century moralist philosophers, particularly in France and England, regarded Machiavellis The Prince as a collection of
cynical precepts for tyrants, and not until the 17th Century was the view of Machiavelli as a teacher of evil questioned by
philosophers.
A. Early 16th Century moralist philosophers, particularly in France and England, regarded Machiavellis The Prince as a collection
of cynical precepts for tyrants, and not until the 17th Century was the view of Machiavelli as a teacher of evil questioned by
philosophers.
1. Reference of him is not correct in this choice. Pronouns can only refer to nouns and other pronouns. In this choice, him
cannot refer to Machiavellis because it is neither noun nor a pronoun. Used in possessive form, this word acts like an adjective
modifying The Prince. It shows that The Prince was written by Machiavelli
ACTIVE VOICE philosophers regarded... opposed to regarded by philosophers
IDIOMATIC regarded as
Machiavelli, referenced for first time, is teacher of evil
B. Machiavellis The Prince was regarded as a collection of cynical precepts for tyrants by early 16th Century moralist
philosophers, particularly in France and England, and not until the 17th Century did philosophers question the view of him as a
teacher of evil.
HIM has no noun... 'possessive poision'. 'Machiavelli's' cannot be the noun referenced by HIM.
C. Early 16th Century moralist philosophers, in France and England particularly, considered Machiavellis The Prince as a
collection of cynical precepts for tyrants, and not until the 17th Century was the view of Machiavelli as a teacher of evil
questioned by philosophers.
CONSIDERED TO BE is correct idiom
D. Early 16th Century moralist philosophers, particularly in France and England, regarded Machiavellis The Prince to be a
collection of cynical precepts for tyrants, and not until the 17th Century philosophers was the view of Machiavelli as a teacher of
evil questioned.
REGARDED AS is correct idiom
E. Machiavellis The Prince was regarded as a collection of cynical precepts for tyrants by early 16th Century moralist
philosophers, particularly in France and England, and not until the 17th Century was the view of him as a teacher of evil
questioned.
HIM has no noun... 'possessive poision'. 'Machiavelli's' cannot be the noun referenced by HIM.
1. Entities in a parallel list must be grammatically parallel to the extent that they maintain the logical meaning of the sentence.
2. We must use idiomatic expressions.
3. Pronouns can only refer to nouns and other pronouns.
jitbec wrote:
Correct Answer: A
Explanation:
Note the idioms regarded as something and consider something. In other words we dont put to be after regard or consider,
and neither do we put as after consider. This information helps us to eliminate C and D. next we turn our attention to word
order. The order in B and E suggests that the precepts were by the moralist philosophers. And so we are left with A. Also note
that
the use of him in B and E is inappropriate as we havent previously mentioned Machiavelli (Machiavellis The Prince is not the
same as Machiavelli!) Note that the correct answer includes the passive voice was questioned by, and so, although the active
voice is usually better than the passive, there are some cases where passives are found in correct answers.
READ SENTENCE COMPLETELY DO NOT SKIP LAST NE ...OR LAST PART OF ANY CHOICE
Yes, this is a good explanation..I had picked B because I ignored the use of him and the second half of A was passive. But now I
can see why A is correct. We got to choose the best option
_________________
Regarded by opponents as ineffective and meddlesome and by supporters as a conserver of life and energy, the fight over the
speed limit continues in our legislatures and on our freeways.
A. Regarded by opponents as ineffective and meddlesome and by supporters as a conserver of life and energy, the fight over the
speed limit continues in our legislatures and on our freeways.
B. Regarded by opponents as ineffective and meddlesome and by supporters as a conserver of life and energy, the speed limit
continues to be fought over in our legislatures and on our freeways.
C. Regarded by opponents as ineffective meddling and by supporters as the conservation of life and energy, the speed limit
continues to be fought over in our legislatures and on our freeways.
D. The fight over the speed limit, regarded by opponents as ineffective and meddlesome and by supporters as a conserver of life
and energy, continues in our legislatures and on our freeways.
E. The fight over the speed limit, a measure regarded by opponents as ineffective and meddlesome and by supporters as a
conserver of life and energy, continues in our legislatures and on our freeways.
[Obscure] Spoiler:
So the way it's structured right now, the modifier is incorrectly placed (as many of you have noticed). It's not the "fight over
the speed limit" that's meddlesome or a conserver of life and energy, it's the speed limit itself. We need to find the answer
choice that correctly attaches the modifier ("Regarded by opponents as ineffective and meddlesome and by supporters as a
conserver of life and energy") to its correct noun ("speed limit"). This way, we can eliminate A for sure. D is also wrong
because of the same reason: incorrectly placed modifier.
C is wrong because there's no parallelism between "ineffective meddling" (verb) and "the conservation of life and energy"
(noun). Also, the use of "the" in bold is unnecessary. "The" suggests that the speed limit is "the one and only" (to quote a
song Very Happy) saver of energy and life, which is obviously not the case - I can think of a million other things that can
save more lives and energy Smile
Now, you guys have successfully narrowed it down to B and E... E is better here because of the use of the passive voice in
B. In general, you should avoid the passive voice in the GMAT, because it's considered wordy. "To be fought over" is also
pretty vague - who fights over it? The official explanation also cites "a pile-up of prepositions" in "to be fought over in".
Rumored, and believed by most, to have committed suicide on stage, folk musician Sixto Rodriguez, a legend to his fans in South
Africa, was actually working construction jobs in Detroit, unaware that his music had influenced so many.
A. Rumored, and believed by most, to have committed suicide on stage, folk musician Sixto Rodriguez, a legend to his fans in
South Africa, was actually working construction jobs in Detroit, unaware that his music had influenced so many. - Correct
B. Rumored and believed by most to have committed suicide on stage, folk musician Sixto Rodriguez actually worked
construction jobs in Detroit; a legend to his fans in South Africa was unaware that his music had influenced so many. - wrong
reference for subject
C. Rumored, and believed by most, that he committed suicide on stage, folk musician Sixto Rodriguez, a legend to his fans in
South Africa, was actually working construction jobs in Detroit, where he was unaware that his music had influenced so many.
-Use of that is wrong, where he was unaware is also wrong
D. Rumored and believed by most to have committed suicide on stage, folk musician Sixto Rodriguez is a legend to his fans in
South Africa; he was actually working construction jobs in Detroit, unaware that his music had influenced so many. - Verb tense
error
E. While he was rumored, and believed by most, to have committed suicide on stage, Sixto Rodriguez was actually working
construction jobs in Detroit, unaware that his music influenced so many. - he was rumored is incorrect
_________________
Apoorv
Rumored, and believed by most, to have committed suicide on stage, folk musician Sixto Rodriguez, a legend to his
fans in South Africa, was actually working construction jobs in Detroit, unaware that his music had influenced so
many.
A. Rumored, and believed by most, to have committed suicide on stage, folk musician Sixto Rodriguez, a legend to his fans in
South Africa, was actually working construction jobs in Detroit, unaware that his music had influenced so many.
B. Rumored and believed by most to have committed suicide on stage, folk musician Sixto Rodriguez actually worked
construction jobs in Detroit; a legend to his fans in South Africa was unaware that his music had influenced so many.
C. Rumored, and believed by most, that he committed suicide on stage, folk musician Sixto Rodriguez, a legend to his fans in
South Africa, was actually working construction jobs in Detroit, where he was unaware that his music had influenced so many.
D. Rumored and believed by most to have committed suicide on stage, folk musician Sixto Rodriguez is a legend to his fans in
South Africa; he was actually working construction jobs in Detroit, unaware that his music had influenced so many.
E. While he was rumored, and believed by most, to have committed suicide on stage, Sixto Rodriguez was actually working
construction jobs in Detroit, unaware that his music had influenced so many.
Vertical Split.
E is incorrect because of While.
C --> incorrect use of commas. It is both rumored and believed.
D --> contrast author wants to point out s missing.
B --> Modifier error. legend to his fans is not modifying Sixto
A rules
Answer choice D illogically uses the past tense verb was after the semicolon.
Official answer:
A.
Explanation:
Answer choice B is guilty of a reference error, as the phrase after the semicolon a legend to his fans does not specific
whether Rodriguez or some other person was that legend.
Answer choice C is wrong because the phrase Rumored that he committed is not a proper modifier for the subject Sixto
Rodriguez.
Answer choice D illogically uses the past tense verb was after the semicolon.
And answer choice E illogically modifies Rodruigez work by saying it happened while he was
rumored to have committed suicide.
Some people believe that gaining wealth is due to luck. However, research from many institutions worldwide
indicates a strong correlation between gaining wealth and high educational levels. Thus research supports the view
that gaining wealth is largely the result of making informed career and business choices.
The reasoning in the argument is most vulnerable to criticism on the grounds that the argument..
A.presumes, without providing justification that only highly educated people make informed career and business
choices. WRONG BECAUSE OF WORD "ONLY".....OTHER THINGS BESIDES EDUCATION COULD ALSO LEAD TO
INFORMED CHOICES....
B.overlooks the possibility that people who make informed career and business choices may nonetheless belong to a
poor family..... IRRELEVANT...
C.presumes, without providing justification, that informed career and business choices are available to everyone.
IRRELEVANT....
D.overlooks the possibility that the same thing may causally contribute both to education and to gaining wealth.
CORRECT....."EDUCATION" MAY NOT LEAD TO "INFORMED CHOICES" BUT SOMETHING ELSE MAY HAVE LED TO
BOTH....
E.does not acknowledge that some people who fail to make informed career and business choices have gained
wealth. IRRELEVANT.....
Hi Mike,
How can we be so sure that the disease will definitely arrive for young children. The word "will" is very strong in
option A.
Thanks, Jai
JUST MOVE UR WAY REVERSE BUT BEFORE THAT READ THE KEYWORD AND SCOPE OF
QUETION....NOW GET THE CONCLUSION AND LINK ORMEMISES TO THE CON;LSION ....SEEING ANY
SCOPE CHANGE OR ANY JUMP ....AND FINALLY JUST GO HAEAD !!WITH A PRETHOUGHT
Public Health Official: After several years of vaccinating all of the citizens of this state for Tacitus
Disease, a highly infectious virus, state hospitals have cut costs by no longer administering this
vaccine, starting at the beginning of this year. A state senator defended the position, arguing that after
several years with zero incidence of the disease in the state, its citizens were no longer at risk. This is a
flawed argument. Our state imports meats and produce from countries with high incidences of
diseases for which our country has vaccines. Three years ago, when we reduced the use of the
Salicetiococcus vaccines, a small outbreak of Salicetiococcus among young children, fortunately
without fatalities, encouraged us to resume use of the vaccines at the previous levels.
The public health officials statements, if true, best support which of the following as a conclusion?
(A) Young children of the state will be at risk for Tacitus Disease.
(B) Some of the meats imported to this state do not have adequate refrigeration during the shipping
process.
(C) Tacitus Disease is a much deadlier disease than Salicetiococcus, and has a correspondingly higher
fatality rate.
(D) No food products produced within the state bear any contaminants that could lead to either
Tacitus Disease or Salicetiococcus.
(E) The cost of providing all citizens of the state with the Tacitus Disease vaccine places an undue
burden on the budget of state health agencies.
One common GMAT CR question type is the "find the conclusion" or "find the inference" question. For a full
discussion of this question type, as well as an explanation of this question, see:
Dear Jai,
I'm happy to respond.
Public Health Official: After several years of vaccinating all of the citizens of this state for Tacitus Disease, a highly
infectious virus, state hospitals have cut costs by no longer administering this vaccine, starting at the beginning of
this year. A state senator defended the position, arguing that after several years with zero incidence of the disease in
the state, its citizens were no longer at risk. This is a flawed argument. Our state imports meats and produce from
countries with high incidences of diseases for which our country has vaccines. Three years ago, when we reduced
the use of the Salicetiococcus vaccines, a small outbreak of Salicetiococcus among young children, fortunately
without fatalities, encouraged us to resume use of the vaccines at the previous levels.
Yes, the word "will" is a strong word, but we absolutely know this to be the case. You see, we know that Tacitus
Disease is "a highly infectious virus," which means people get it very easily. It appears that the only reason Tacitus
Disease hasn't be active for years is that the entire population has been vaccinated. Keep in mind, people who are
vaccinated can carry the virus, but they simply don't get sick from it. In all likelihood, the vast majority of members
of this population are passive carriers of the virus, so in all likelihood, the virus is still present in the population. If the
authorities stop the vaccinations, then the children born after that time will be without any protection against this
highly infectious disease. We can't say for sure that the children will get the disease, but it would seem that the
probability is very high. We can say for sure that they are at risk. Any time someone is exposed to any danger
without sufficient protection, by definition, they are "at risk." Infectious disease, no vaccination --- that's "at risk."
(A) The first is a principle that the argument relies on and the second is a conclusion that can be drawn from the
first.
The reliance of navigation system is a fact and NOT a principle. A PRINCIPLE is a rule/law or moral
tenet or some other guideline that is a basis of decision... NOT A PRINCIPLE! OUT
(B) The first is a fact that argument relies on and the second is a conclusion that must be drawn from this argument.
The first is a FACT indeed BUT it is not what the argument relies on... Try to remove that particular
sentence and see if the argument will still hold...
Modern navigation systems, which are found in most of todays commercial aircraft, are made with
low-power circuitry, which is more susceptible to interference than the vacuum-tube circuitry found in
older planes. [b]During landing, navigation systems receive radio signals from the airport to guide the
plane to the runway. Recently, one plane with low-power circuitry veered off course during landing, its dials
dimming, when a passenger turned on a laptop computer. Clearly, modern aircraft navigation systems are
being put at risk by the electronic devices that passengers carry on board, such as cassette players and
laptop computers.
[/b]
(C) The first acknowledges a consideration that supports that main position; the second is that conclusion.
(D) The first is an evidence that supports the conclusion, the second is that conclusion.
This is a fact under consideration. The evidence is not this one but the
turning on of the laptop...
(E) The first is a principle that is necessary for this argument, the second is a conclusion that could be drawn from
this argument.
Like in A, it is NOT A PRINCIPLE
Answer: C
Hi Friends,
I found some mistakes in the question.. I have posted the correct version with explanation..
The Republic of Estonia is separated from Finland by the narrow Gulf of Finland. The Estonian people are genetically
similar the Finnish people. Estonians also speak a language that is very closely related to Finnish. It is likely that,
many centuries ago, residents of what is now Estonia migrated to what is now Finland and established settlements
there. Which of the following, if true, does the least to undermine the author's explanation for the similarities given?
A. The earliest settlements found in Finland that exhibit the cultural influences common to Estonians and Finns
predate the earliest settlements found in Estonia that exhibit these traits.
B. At the time at which the residents of present-day Estonia are believed to have migrated to present-day Finland,
travel by boat across the Gulf of Finland was not possible.
C. Early Estonian artifacts contain drawings of animals that were native only to Finland at the time the migrat!
ion was believed to have taken place.
D. The traits common to the peoples of Finland and Estonia are also shared by the population of present day
Sweden.
E. The Norwegian people are more similar to the Finnish people than are the Estonians.
Correct Answer: D
The author's argument concludes that early Finns migrated from Estonia, based on similarities between the two
cultures. Answer choices that are incorrect will weaken this conclusion, likely by giving an alternate explanation for
why these cultures share traits. Choice (D) does not weaken the author's conclusion, and thus is the correct answer,
because the fact that these traits are also shared by the people of Sweden does not have any effect on the proposed
migration from Estonia to Finland; perhaps the path was Sweden to Estonia to Finland, or Estonia to Finland to
Sweden. Choice (A) is incorrect because it gives some evidence that the migration was from Finland to Estonia, not
in the other direction as the author believes. Choice (B) is incorrect because it eliminates one possible means by
which the Estonians could have migrated to Finland. Choice (C), like choice (A), gives evidence for a migration from
Finland to Estonia. Choice (E) presents the possibility of a stronger link between Norwegians and Finns than Finns
and Estonians, weakening the likelihood that the Finnish people migrated from Estonia.
(A)The clinical checkups were intended for the early identification of women who are at risk of post-natal
depression, and not for premature labor prevention.
Possible choice. Come back to it later.
It's true that even I was vacillating between A and C (30-70) a bit. However, it's possible for a process to achieve
more than it's intended for. Moreover, maybe the very act of identifying patients who're likely to be subjected to
the post-natal depression causes the reduction in preterm labor, who knows. Perhaps the women who fall in this
category are the very women who are subjected to preterm labor. After identification, they could get preventive
treatment and counseling, resulting in reduction of preterm labor.
(B)Four years ago, a dozen hospitals in Country X were given new equipment to facilitate the extraction of new-
born babies, thus lowering significantly the number of still births.
Dozen hospitals may just be a small percentage of hospitals in country X. If country X has million hospitals, we
can't gauge much on this statistics from what happened in just few dozens of those million hospitals.
(C)The cessation of hostilities in Country X coincided with the introduction of clinics offering checkups to pregnant
women, and women in war zones are more prone to go into early labor.
This is reasonable. Because the reduction may have caused as a positive effect of cessation of war and hostilities.
(D)Eight years ago, the UN launched a campaign in Country X promoting the use of medicine to reduce the cases of
infections of the urinary tract, following which the percentage of woman going into premature labor was reduced
by 30%.
The reduction started 4 years ago, not 8.
(E)Women throughout the world, including those in Country X, go into premature labor, inevitably resulting in a
certain percentage of still births.
This is an info about preterm labor that doesn't affect the argument in anyway. Out of scope.
Go back to A.
Source: Master GMAT.
Official Explanation:
Originally a trading port, Hong-Kong, currently estimated to be fourth most densely-populated region in the world,
turned into a dependent territory of Great Britain in 1842 and was transferred to China on July 1st, 1997.
(1) Originally a trading port, Hong-Kong, currently estimated to be fourth most densely-populated region in the
world,
(2) Hong-Kong, currently estimated to be fourth most densely-populated region in the world, was originally a trading
port that
(3) Hong-Kong, currently estimated to be fourth in the list of the most densely-populated regions in the world, was
originally a trading port that
(4) Hong-Kong, currently estimated to be fourth in the list of most densely-populated regions in the world, was
originally a trading port that
(5) Originally a trading port estimated to be the fourth most densely-populated region in the world, Hong-Kong
Originally a trading port, Hong-Kong, currently estimated to be fourth most densely-populated region in the world,
turned into a dependent territory of Great Britain in 1842 and was transferred to China on July 1st, 1997.
This sentence is giving some information about Hong Kong. These information include:
1. Hong Kong is originally a trading port.
2. Currently it is estimated to be the fourth most densely-populated region in the world.
3. It turned into a dependent territory of Greta Britain in 1842.
4. It was transferred to China on July 1st, 1997.
On carefully reading this sentence, we notice that all information given about Honk Kong are independent. They are
not related anyway except for the fact that all these information are about the same place Honk Kong. Since, choice
A is incorrect for the reason already discussed in the thread, we need to find an answer choice that provides all these
information.
Choice C: Hong-Kong, currently estimated to be fourth in the list of the most densely-populated regions in the world,
was originally a trading port that
This is the correct answer choice and the explanation for the same has already been provided in previous
discussions. This choice does not change the meaning of the sentence in any way. Except for choice E, all the answer
choices convey the intended meaning.
Choice C presents all the information that we get from the original sentence. In this clause that turned into
dependent territory, that stands for a trading port and what is that trading port? Honk Kong is that trading
port. So its telling the same thing in a different way. All the information remains the same. So there is no change in
meaning in choice C.
HERE A IT REFERNCE WITH SEMICOLON WOULD HAVE REFERRED TO A 2009 STUDY .IF THE SEMI
COLON WAS USED .SO USE A COLON AS IT EXPLAINS TH ETHE THAT CLAUSE AND IF COMMA
WAS USED THEN IT SHOULD BE INDEPENDENT CLAUSE WHICH IT IS NOT SO NO COMMA
REQUIRED AS SMAL AND OLD IS ONE THING .AS THERE IS A USAGE OF A SEPARATE ANSD FOR IT
A 2009 study from the California State Housing Authority concluded that conversion from ownership to rental
properties has often been difficult; it has been more common for some townhouses and other attached homes that
are relatively small and old, located in central cities.
A. difficult; it has been more common for some townhouses and other attached homes that are relatively small and
old, located in central cities.
B. difficult; it has been more common for some townhouses and other attached homes that are relatively small,
old, and that are located in central cities.
C. difficult; it has been more common for some townhouses and other attached homes, which are relatively small
and old, and located in central cities.
D. difficult: It has been more common for some townhouses and other attached homes that are relatively small and
old and located in central cities.
E. difficult: It has been more common for some townhouses and other attached homes that are relatively small and
old, and located in central cities.
Contemporary ballet is a form of dance influenced by both classical ballet and modern dance, employing the
fundamental technique and body control of classical ballet but permits a greater range of movement and may not
adhere to the strict body lines or turnout that permeate classical ballet
A. Contemporary ballet is a form of dance influenced both by classical ballet and modern dance, employing the
fundamental technique and body control of classical ballet, but permits a greater range of movement and may not
adhere to the strict body lines or turnout that permeate classical ballet
Classical Ballet is singular and so "permeate" must reflect that. It should be "permeates"
B. Contemporary ballet is a form of dance influenced both by classical ballet and by modern dance, employing the
fundamental technique and body control of classical ballet but permitting a greater range of movement than classical
ballet, and may not adhere to the strict body lines and turnout that permeates classical ballet
There is a problem with parallelism. Contemporary ballet is a form of dance, employing... but permitting... and not
adhere. "Adhere" should be "adhering"
C.Contemporary ballet is a form of dance influenced both by classical ballet and by modern dance, employing the
fundamental technique and body control of classical ballet, but permitting a greater range of movement and may not
be adhering to the strict body lines or turnout that permeate classical ballet
C. keeps the parallel structure of employing... permitting... and adhering... but fails to have "permeate" reflect the
singular "classical ballet". It should be "permeates".
D. Influenced both by classical ballet and by modern dance, contemporary ballet is a form of a dance employing the
fundamental technique and body control of classical ballet, but permitting a greater range of movement and not
adhering to the strict body lines or turnout that permeates classical ballet ---
E. Employing the fundamental technique and body control of classical ballet, but permitting a greater range of
movement , contemporary ballet is a form of dance that may not adhere to the strict body lines and turnout that
permeate classical ballet
The same error with "permeate" occurs.
OA: D
OE:
A.---- the verb should be permeates
B.---- bodylines and turn out that permeates SV error
C.--- may not be adhering is a clause; parallelism broken
D.correct choice
E. The stress is more on not adhering to the strict body lines and turnout
I did this ques in 2 min just by focusing on the singular verb permeates. Since A or B is
used..singular word should be used...hence D.
I dont know if it was the correct approach.
THIS ONE TOOK TIME BUT I GOT ITT CORRECT .MIND IT CORRECT
just see what are the list ...and how can they be connected ...........FANBOYS....any one
of them can beused ....do not restrict with " and " only
Hurricane resistant classification is given by the government to those structures that use construction materials
such as solid concrete, steel, reinforced brick, cement or pyramid shaped roof with slopes of no less than 35
degrees from horizontal and no overhangs of any kind.
A) reinforced brick, cement or pyramid shaped roof with slopes of no less than 35 degrees from horizontal and no
overhangs of any kind
B) reinforced brick, or cement, or if the structures have pyramid shaped roof with slopes of no less than 35
degrees from horizontal and no overhangs of any kind
C) reinforced brick, and cement, or if the structures have pyramid shaped roof with slopes of no less than 35
degrees from horizontal or no overhangs of any kind
D) reinforced brick, cement, if the structures have pyramid shaped roof with slopes of no less than 35 degrees
from horizontal, and no overhangs of any kind
E) reinforced brick, cement, pyramid shaped roof with slopes of no less than 35 degrees from horizontal, or no
overhangs of any kind
I was hoping to see little confusion over this after I got this wrong after a whisker. But, here is what I have in OE.
OE
What does the sentence mean - The sentence states two conditions under which the government classified a
structure as hurricane resistant.
1: Either the construction materials should be solid concrete, steel, or reinforced brick
2: Or the structure should have pyramid shaped roof with slopes of certain slope and no overhangs of any kind.
What are the errors in the original sentence The original sentence has parallelism error since the identified
list is not correct. There are indeed two lists in this sentence that have been combined into one.. First one lists out
the construction materials and second one gives the conditions that a structure should meet in order to be classified
as hurricane resistant
List 1 - structures that use construction materials such as solid concrete, steel, reinforced brick, cement
List 2 structures that use construction materials or structures that have pyramid shaped roof
I am giving choice analysis for only C here as it is the only one which is real contender. Choice C is rendered bad
with his explanation: This choice changes the intended meaning since it uses the connector or to connect the two
features of the pyramid shaped roofs. This is because the original sentence used and to connect these elements
thereby implying that both these conditions should be satisfied for the structure to be classified as hurricane
resistant.
What I really feel is that pyramid shaped roof should have got some reference(like that/which) back to structures as
explained in OE, though not present in OA. According to me, correct list would have been better justified with a
conjunction/relative pronoun as in the sentence below.
Hurricane resistant classification is given by the government to those structures that use construction materials such
as solid concrete, steel, reinforced brick,or cement or that pyramid shaped roof with slopes of no less than 35
degrees from horizontal and no overhangs of any kind
Contemporary accounts of the life of Charlemagne, crowned emperor by the pope in 800, show that the founder of
the Carolingian renaissance in literature and the arts was himself an illiterate driven by his desire for a civilized state
to reform education in his kingdom.
a) that the founder of the Carolingian renaissance in literature and the arts was himself an illiterate driven by his
desire
b) that the founder of the Carolingian renaissance in literature as well as in the arts was himself an illiterate and also
driven by his desire
c) that the founder of the Carolingian renaissance in literature and the arts was himself an illiterate and that he was
driven by his desire
d) that the founder of the Carolingian renaissance in literature as well as in the arts was himself an illiterate and that
he was driven by his desire
e) that the founder of the Carolingian renaissance in literature and the arts was himself an illiterate and that his
desire drove him \
OA : A
OE: A glance at the answer choices suggests that the issue here involves the participle driven. The sentence gives the
surprising detail that a person interested in literature was illiterate, and then explains why an illiterate would have such an
interest. As such, the participle driven is an integral description of the word illiterate, not of the man in general.
Therefore, the participle must stand as it is and choice (A) is correct. Each other choice separates illiterate from driven,
altering the meaning of the sentence.
The phrase introduced by "as well as" will modify the earlier word (mayor in
this case), but it does not compound the subjects (as the word "and" would
do).
Americans give pride of place to the value of individual liberty, and we find especially unintelligible the infliction of
suffering on the innocent.
(A)....
(B) especial unintelligible the infliction of suffering
(C) especially unintelligible sufferage that is inflicted
(D) especially unintelligible the suffering that is inflicted
(E) especially unintelligible the inflicting to suffer
The OE reads:
D - worng because the sentence changes the meaning. It's not that the Americans find the suffering unintelligible;
rather Americans find the infliction of suffering unintelligible
C - I made a mistake while keying in (Sorry Professor) It should have been suffrage - which means right to vote.
_________________
(A) The Sri-NuanSri-Nuan dance and Teut-Teung dance are an inherent part of Thai culture, and along with others
such as The Farmer's Dance, developed by the Ministry of agriculture, constitutes the main dramatic art form of
Thailand
(B) The Sri-NuanSri-Nuan dance as well as Teut-Teung dance are an inherent part of Thai culture, and along with
others such as The Farmer's Dance, developed by the Ministry of agriculture, constitute the main dramatic art form
of Thailand
Wrong, Sing subject <--> Plural verb
(C) The Sri-NuanSri-Nuan dance and Teut-Teung dance are an inherent part of Thai culture, and along with The
Farmer's Dance, developed by the Ministry of agriculture, constitute the main dramatic art form of Thailand
Wrong, change of meaning : the Farmer's Dance is just an example in the original sentence ( such as ..)
(D) An inherent part of Thai culture, the Sri-NuanSri-Nuan, Teut-Teung as well as other dances such as The Farmer's
Dance, developed by the Ministry of agriculture, constitute the main Thai dramatic art form
Wrong , Misplaced modifier
(E) The Sri-NuanSri-Nuan and Teut-Teung dances, both inherent parts of Thai culture, and a dance developed by the
Ministry of Agriculture named The Farmer's Dance, constitute, along with others, the main dramatic art form of
Thailand
Correct
First, let's think about what the sentence is trying to say without pronouns - it's trying to point out a problem and its
obverse: detect a condition when the condition is present vs. indicate that the condition is present when the condition is
not present.
A) does a fine job with the first bit (detect a condition when it is present) but messes up the second bit "indicate that there
is one when it is not" - indicate there is one what? One condition? It sounds like it's saying something like "one condition
exists." The point is to indicate the presence of the condition in a particular circumstance, not its mere existence on the
planet.
C) does a fine job with the first bit (detect a condition when it is present) and the second bit (indicate it is present when it is
not). Fine.
D) muddies the meaning by moving "when" to the front (B does this too). It sounds like the test just can't detect the timing
of the condition, as though, sometimes I have cancer and sometimes I don't and the test can't tell when I do or don't.
Really, the test is failing to detect the condition itself, not the timing of the condition. See E for the problem at the end of
this choice.
E) "indicate its presence when it is not" is incorrect - that translates as "indicate its presence when it is not presence (last
word implied)" which doesn't make any sense. We need to say "indicate that it is present when it is not present (last word
implied)." Also "the presence of a condition when it is there" is wordy. "a condition when it is present" is much cleaner.
Stacey Koprince
Instructor
Director of Online Community